You are on page 1of 115

Medicbyte

THEME: Diagnosis Of Nepotistic Lesion


Option:
A. Craniopharyngioma
B. Pineal adenoma
C. Astrocytoma
D. Lymphoma
E. Pituitary adenoma
F. Nephroblastoma
G. Ganglioneuroma
H. Neuroblastoma
I. Carcinoid tumour
J. Thyroid follicular carcinoma
K. Adrenal malignant infiltration
L. Medullary thyroid carcinoma
Instruction
For each patient below choose the SINGLE most likely diagnosis form the above list
of options. Each Options may be used once, more than once, or not at all.

Q-1. A 35-year-old women who was diagnosed as having coeliac disease at


the age of 12 years has had good control of her abdominal symptoms for
a number of year. She presents with a recent his history of weight loss,
diarrhea & abdominal pain.
Q-2. A 15 year-old girl whose weight is on the 50 th centile, has double
vision.
Q-3. A 32 year-old man develops pigmentation of his skin this looks very
much like a sun than. He has also been very lethargic & depressed.
Q-4. A young man is admitted acutely with vomiting. His BP id 160/100 &
heart rate 38.
Q-5. A young woman complains of flushing after drinking coffee, diarrhea,
swollen ankles & intermittent wheeze.

Jen & Adil

Medicbyte
THEME: Bacterial Infections
Options:
A. Streptococcus Pneumoniae
B. Escherichia Coli.
C. Mycobacterium Tuberculosis
D. Staphylococcus Viridans
E. Atypical Pneumoniae
F. Leptospira Canicola
G. Staphylococcus Aureus.
H. Staphylococcus Faccalis
I. Branhamella Catarrhalis
J. Neisseria Meningitidis
K. Borrelia borgdorferi
Instructions
For each patient below choose the SINGLE most likely diagnosis form the above list
of options. Each Options may be used once, more than once, or not at all.

Q-6. A 25-year-old man has had problems with a recent middle ear infection
He is found unconscious.
Q-7. A two week old with a congenital anomaly of the rental tract presents
with extreme with extreme lethargy, tachycardia & pyrexia.
Q-8. An Intravenous drug abuser who regularly injects to get a high has a
temperature of 40 degree Celsius.
Q-9. A 35-year-old Pakistani Lady with a five month history of persistent
headaches.
Q-10. A young man returns from a camping & presents with arthralgia
enlarged lymph nodes & a rash consisting of multiple ring lesions.
Q-11. An intravenous drug abuser develops a dry cough & night sweats. A
Mantoux test is negative.

Jen & Adil

Medicbyte
THEME: Clotting Disorders:
Options:
A. Heparin
B. Warfarin
C. Vitamin K
D. Factor VIII concentrate
E. Desmopressin
F. Fresh Frozen Plasma
G. Factor-Ten
H. Oral Ferrous Sulphate
I. Blood transfusion
J. Intra muscular
K. Nothing is required
L. Steroids
Instructions:
For each patient below choose the SINGLE most likely diagnosis form the above list
of options. Each Options may be used once, more than once, or not at all.

Q-12. A lady who has a heart valve has found that she has a positive
pregnancy test. She needs advice about anticoagulation.
Q-13. A 65-year-old has liver failure & bleeding varies. The surgeon has
decided to take her to theatre to stem the blood loss.
Q-14. A 10-year-old girl is found to have prolonged time serum factor eight
levels are found to be normal.
Q-15. A 12-year-old boy with thalassaemia is breathless with hemoglobin of
8.0.
Q-16. During recovery from a severe pneumonia a young boy develops rapid
onset self-limiting purpura.
Q-17. A young women presents with 10-years history of intermittent
bleeding purpura & recurrent nosebleeds.

Jen & Adil

Medicbyte
THEME: Neurological Signs:
Options:
A. Extrapyramidal signs
B. Cerebellar signs
C. Lower motor neuron signs
D. Upper motor neuron signs
E. Peripheral neuropathy neuropathies signs
F. Mononeuropathy signs
G. Mixed upper & lower motor neuron signs
H. Proximal myopathy
I. Diminishing reflexes
J. Descending loss sensation
K. Dissociated sensory loss
L. Circumoral paraesthesia
Instruction:
For each patient below choose the SINGLE most likely diagnosis form the above list
of options. Each Options may be used once, more than once, or not at all.

Q-18. A 35-year-old women has been diagnosed with an acoustic neuroma.


Q-19. A 12-year-old boy has Wilsons disease.
Q-20. A 25-year-old lady has respiratory difficulty related to Guillain Barre
syndrome
Q-21. A 6-year-old girl has been eating old paint peeling of the walls of her
bedroom. Serum lead has been detected.
Q-22. A 8-year-old boy has arrived from India, he has never been
vaccinated against polio & has now developed the illness.

Jen & Adil

Medicbyte
THEME: Blood Films:
Options:
A Macrocytosis alone
B Microcytic anaemia.
C Dimorphic blood film (macrocytes & microcytes)
D Normochromic normocytic anaemia
E Howell jolly bodies
F Ring Sideroblasts
G Lucoerythroblasts
H Microangiopathic blood film
I Hairy cells
J Profuse mature lymphocytes and smear cells
K Eosinophilia
L Atypical lymphocytes
Instruction:
For each patient below choose the SINGLE most likely diagnosis form the above list of
options. Each Option may be used once, more than once, or not at all.

Q-23. A 6-year-old boy has weight loss, loose stool rickets & lethargy
Q-24. A 2-year-old has been mainly on cows milk since 6-months of age. He does
not like eating solids & prefers bottled milk.
Q-25. A 46-year-old factory worker has been recently taking down lead piping from
an old factory site.
Q-26. A 8-year-old girl with sickle cell anaemia has had numerous sickle crises
Q-27. A pt has late stage multiple myeloma.
Q-28. A young woman develops diffuse intravascular coagulation & jaundice.
Q-29. A 12-year-old girl has a sore throat & exudates on her palate, her neck
glands are swollen
Q-30. A pregnant women is noted to be both iron & Folate deficient.
Q-31. A traveler passes long worms in his stool
Q-32. A 17-year-old girl has menorrhagia.
Q-33. A 33-year-old lady has longstanding emphysema.

Jen & Adil

Medicbyte
THEME; D/D of Haematuria:
Options:
A Systemic lupus erythematosus.
B Haemolytic Uraemic Syndrome
C Acute Lymphoblastic Leukemia
D Neuroblastoma
E Nephroblastoma
F Bartters Syndrome
G Juvenile Chronic Arthritis
H Henoch-Schonlein Purpura
I Good Pasteurs Syndrome
J IgA Nephropathy.
K Prostatic Carcinoma
L Polycystic Kidney Disease
Instruction
For each patient below choose the SINGLE most likely diagnosis form the above list
of options. Each Option may be used once, more than once, or not at all.

Q-34. A hypertensive 48-year-old lady has been on hydralazine therapy


Q-35. A 25-year has been on a camping holiday where he ate a beef burger
& then became actual unwell.
Q-36. A 5-year-old has had nosebleeds for the last four days. He has been
very tired of late & has lost weight. Recently he has developed
haematuria.
Q-37. A non-blanching maculo-papular rash is noted on the flexor
compartments of the lower limbs of a twelve-year-old boy.
Q-38. A young man complains of passing blood in his urine. This occurs,
every time he has a respiratory infection.

Jen & Adil

Medicbyte
THEME: Blood Gas Analysis.
Options:
A Metabolic Alkalosis
B Metabolic Acidosis
C Respiratory Alkalosis
D Respiratory Acidosis
E Normal compensated pH

Instructions
For each patient below choose the SINGLE most relevant investigation Options form
the above list. Each Options may be used once, more than once, or not at all.

Q-39. A 25 years old lady arrived hysterical into accident & emergency. Her
arms & legs rigid and she is screaming.
Q-40.A 2-month-old baby has projectile vomiting.
Q-41. A 30-year-old has overdosed on salicylate-based medication.
Q-42. Long-term Phenformin has been used in an elderly patient with typeII diabetes mellitus.
Q-43. A 50-year-old lady has been taking long terms Frusemide (loop
diuretic) for heart failure.

Jen & Adil

Medicbyte
THEME: Investigations respiratory disease
Options:
A Bronchoalveolar lavage
B Serum precipitins & high resolution CT chest scan
C Mantoux Test
D Arterial Blood Gases
E Serum Ca++ ACE & transbronchial biopsy
F Sputum Culture
G Echocardiography
H Fibreoptic bronchoscopy
I Flow Volume Loop
J Seek immediate expert advice
K ECG, ABGs, & Ventilation Perfusion Scan
Instructions
For each patient below choose the SINGLE most relevant investigation Options form the
above list. Each Options may be used once, more than once, or not at all.

Q-44. A 62-year-old lifelong smoker has a dry cough, weight loss & haemoptysis.
CXR & CT show right middle lobe collapse.
Q-45. A 54-year-old farmer has intermittent dry cough, fever & dyspnoea,
progressive weight loss and fine crepitations on auscultation.
Q-46. A 25-year-old male develops sudden onset dyspnoea, sharp chest pain on
inspiration & haemoptysis whilst straining at stool. He had cholecystectomy
10 days previously. He rapidly becomes cyanosed & loses consciousness. On
examination his BP is 72 / 50 Pulse 156 and thready with a loud pulmonary
heart sound. His JVP is raised by 6 cm.
Q-47. A 37-year-old depressed Nigerian woman develops clubbing, erythema
nodosum, iritis, dyspnoea & lymphadenopathy on her CXR.
Q-48. A 47-year-old Caucasian childminder in Glasgow present with dry cough,
night sweats & haemoptysis.

Jen & Adil

Medicbyte
THEME: Interpretation of Lung Function Tests:
Option:
A. FEV-1 / FVS ratio 90%
Total lung capacity
reduced by 30 %.
considerably reduced
transfer factor.
B. FEV-1 /FVC ratio 60% no
reversibility to
bronchodilators, gas
trapping, hyperinflation &
reduced transfer factor.
C. Normal pulmonary
function tests
D. FEV-1 /FVC ratio 89%
E. FEV-1 /FVC ratio 85%
Globally reduced lung
volumes. Reduced
inspiratory & expiratory
mouth pressure.
F. Reduced transfer factor
alone.

G. Inspiratory & expiratory


fixed obstruction on glow
volume loop
H. Spirometry, lung volumes
& inccorected transfer
factor 50% of predicted,
Corrected transfer factor
almost normal.
I. Increased transfer factor
alone.
J. FEV-1 /FVC ratio of
58%:25% reversibility to
bronchodilators, gas
trapping, normal transfer
factor.
K. Desaturation &
tachycardia on exercise
testing.
L. FEV-1 < I liter.

Instruction:
For each patient below choose the SINGLE most relevant investigation Options form
the above list. Each Option may be used once, more than once, or not at all.

Q-49. Acute exacerbation of asthma


Q-50. Emphysema
Q-51. Pneumonectomy
Q-52. Neuromucular disease (e.g. Myasthenia)
Q-53. Interstitial lung disease.

Jen & Adil

Medicbyte
THEME: Investigation Of Joint Disease:
A. Microscopy & culture of

urethral disease. ESR & Joint


X-Ray
B. Rheumatoid Factor (RF),

joint X-Ray, examination for


extra-articular signs.
C. Joint aspiration, swab &

blood cultures.
D. Clinical history alone, joint X-

Ray may help.


E. Joint aspiration & microscopy

F. Sigmoidoscopy, biopsy &

barium enema.
G. Pelvic spring test
H. Thomas test.
I. Anti double stranded DNA

antibody & complement.


J. Magnetic Resonance

Imaging.
K. Isotope Bone-scan.
L. Spinal X-Ray & HLA profile.

for monosodium urate


crystals.
Instruction:

For each patient below choose the SINGLE most relevant investigation Options form the
above list. Each Options may be used once, more than once, or not at all.

Q-54. A man presents with a single, swollen, red-hot knee. A small


laceration is noted on his foot.
Q-55. A 38-year-old women presents with bilateral tender swellings of the
metacarpophalyngeal joints of the hands, nodules on one elbow & dry
eyes. Her symptoms are worse in the morning.
Q-56. A 52-year-old lady presents with symmetrical distal inter-phalangeal
joint of his left foot.
Q-57.A man Presents with excruciating pain and swelling of the first
meatatarsophalangeal joint.
Q-58. A 25-year-old male returns from a holiday in. Thailand He presents
one month later with dysuria, conjunctivitis & a swollen painful knee

Jen & Adil

Medicbyte
THEME Recognition of causes of jaundice:
Option:
A. Haemolytic Anaemia.
B. Leptospirosis
C. Halothane
D. Gallstones
E. Hepatitis B associated with Primary
hepatocellular carcinoma.
F. Pancreatic Carcinoma
G. Primary Biliary Cirrhosis
H. Chronic active hepatitis
I. Gilberts Syndrome
J. Alcoholic Hepatitis
K. Paracetamol overdose
L. Sclerosing Cholangitis
Instruction
For each patient below choose the SINGLE most relevant investigation Options form the
above list. Each Option may be used once, more than once, or not at all.

Q-59. A 45-year-old woman has skin pigmentation, pruritis, dark urine &
hepatosplenomegaly,. She develops jaundice 5 year after onset.
Q-60. A 62-year-old man presents with jaundice, hepatomegaly, nocturnal
abdominal pain radiating through to the back & weight loss.
Q-61. A 50-year Asian man develops jaundice; right upper quadrant pain,
weakness weight loss, proximal myopathy and depression.
Q-62. A 45-year-old vagrant man develops deep jaundice, abdominal pain,
hypoglycemia, fever and increased prothrombin time. Liver biopsy
shows Mallorys hyaline and collagen deposition.
Q-63. A 32-year-old man presents with jaundice fever mouth ulceration,
blood & mucous per rectum and pyoderma gangrenosum.

Jen & Adil

Medicbyte
THEME Investigation Of Chest Pain
Options:
A. Rectal Examination, Faecal Occult Blood
B. ECG.
C. Arterial Blood Gases
D. Ventilation Perfusion Scan
E. Treadmill Exercise Test.
F. Seek immediate experts advice.
G.Endoscopy
H. CXR
I. Sputum culture
J. Bronchoscopy
K. 2 minute hyperventilation test
L. Coronary Angiography.
Instruction:
For each patient below choose the SINGLE most relevant investigation Options form
the above list. Each Option may be used once, more than once, or not at all.

Q-64 A 50-year-old male complains of intermittent exertional chest pain


radiating to the jaw & left shoulder, relieved by rest An ECG is normal.
Eating can also precipitate his symptoms
Q-65 A 44-year-old diabetic develops central crushing chest pain lasting over
30 minutes & associated with nausea & Vomiting.
Q-66 Epigastric pain & anaemia are documents in a 49-year-old overweight
woman on treatment for joint disease.
Q-67 A 20-year-old man presents with sudden onset pleuritic chest pain &
dyspnoea. He is 182 cm tall & has smoked for 6 year. There is no other
medical history.
Q-68 A 58-year-old chronically hypertensive man is admitted with searing
central chest pain radiating through to the interscapular region.

Jen & Adil

Medicbyte
THEM: Recognition of psychiatric disease.
Options:
A. Bulaemia Nervosa
B. Obsessional Compulsive disorder
C. Phobia
D. Acute confusional state
E. Schizophrenia.
F. Depression with psychomotor retrdation
G. Conversion
H. Cyelothymic personality
I. Hyperventilation syndrome
J. Anorexia Nervosa.
K. Mania
L. Hypomania
Q-69. A mother notices is worried about her 26-year-old son. He seems to jump
from 1 topic to another, make up words, laugh at inappropriate things & talk
to someone imaginary. He has blacked out his windows & complains that
someone else is controlling his thoughts.
Q-70. A 40-year-old man presents to his GP with headaches, insomnia & weight
gain. He is slow & sluggish when he speaks. He describes a loss of interest in
everything a feeling of worthlessness & often cries alone
Q-71. A 23-yr presents with an episode of generalized paraesthesia, flapping
limbs, loss of urinary continence & abnormal posturing of the hands. These
episodes are preceded by chest tightness and difficulty swallowing. Recently
she has been feeling that parts of her are made of cotton wool.
Q-72. A young woman is arrested for inappropriate advances towards a customer at
the supermarket. The following is taken from the policemans report. so
chatty she couldnt get her words out fast enough easily distractible
seemed to be taking to someone in her cell
Q-73. A wife cannot cope with her husbands moods. Her description is hes so
labile he goes from being full of energy & happy one day to being so
pessimistic about everything, saying everythings pointless the next

Jen & Adil

Medicbyte

THEME Bacterial Infections:


Options:
A. Streptococcus Pneumoniae.
B. Escherichia Coli
C. Mycobacterium Tuberculosis.
D. Staphylococcus Faecalis
E. Legionella Pneumophilia
F. Leptospirosis Icterohaemorrhagiae
G. Staphylococcus Aureus.
H. Staphylococcus Epidermidis.
I. Neisseria Gonococcus
J. Branhamella Catarrhalis
K. Borrelia Borgdorferi
L. Chlamydia Psittaci
Instructions
For each patient below choose the SINGLE most relevant investigation Options form
the above list. Each Option may be used once, more than once, or not at all.

Q-74. A 14-year-old girl with sickle cell disease presents moribund.


Q-75. A 45-year-old professor of medicine returned recently from a medical
convention in a tropical country. He had left so warm that he never
switched the air conditioning off & took frequent showers.
Q-76. After breaking out of prison, a prisoner was caught in the nearby
sewage system.
Q-77. A 65-year man presents with fever to the accident & emergency
department. His liver is enlarged. He keeps exotic birds in the back
garden as a hobby.
Q-78 A 6-year-old has nephrotic syndrome & presents with severe
abdominal pain.

Jen & Adil

Medicbyte
THEME Types of Epilepsy:
Options:
A. Partial Seizures
B. Generalized epilepsy
C. Myoclonic jerks
D. Infantile spasms
E. Gestault Seizures
F. Benign Rolandic Epilepsy
G. Petit mal Epilepsy
H. Fifth day Fits
I. Febrile Convulsions
J. Hypocalacmic fits
K. Focal with secondary generalized fits

L. Drug induced seizure

Instructions
For each patient below choose the SINGLE most type of epilepsy Options form the
above list. Each Option may be used once, more than once, or not at all.

Q-79. A 14-year-old girl who has been academically brilliant, lately has had
difficulty with her school work. She has been caught daydreaming in
class on a number of occasions.
Q-80. A 6-year-old girl was rushed to hospital with a fit involved all her
upper & lower limbs. She had an upper respiratory infection a few days
ago. On examination she has a red throat & is pyrexial.
Q-81. A 12-year-old girl was found in bed in the early hours having left
sided jerks. She was also salivating from the side of the mouth &
seemed unresponsive for a few minutes.
Q-82. A newborn baby is born to a mother of Pakistani origin. The baby has
a generalized seizure involving her arm & limbs.
Q-83. A six month old baby has jerking episodes lasting a few minutes at a
time. During these episodes the baby is seen to flex his arm & legs &
the head also flexes. The child has three to four episodes every hour.

Jen & Adil

Medicbyte
THEME

Syndromes:
Options:
A. Downs syndrome
B. Turners syndrome
C. Edwards syndrome
D. Pierre Robin syndrome
E. Alports syndrome
F. Klinefletter syndrome
G. Prader Willi syndrome
H. Willaims syndrome
I. Duchenne Muscular Dystrophy
J. Angelmans syndrome
K. Storage disease
L. Tuberous sclerosis

Instructions
For each patient below choose the SINGLE most relevant investigation Options form
the above list. Each Options may be used once, more than once, or not at all.

Q-84. A12-year-old girl has radio-femoral delays, is short & apubertal.


Q-85 . A 2-year-old boy has a pan systolic murmur at the left sternal edge
& a liver that is displaced by three finger breathes, he is also breathless
with feeds.
Q-86 . A newborn baby is born with rocker bottom feet.
Q-87. A baby has hypergonadotrophic hypogonoadism.

Jen & Adil

Medicbyte
THEME Diagnostic Neurological Signs
Options:
A. Friedrichs Ataxia
B. Cerebellar ataxia
C. Dystonia.
D. Mytonia.
E. Muscular dystrophy.
F. Hepatolenticular degeneration
G. Myoclonus.
H. Parietal lobe problems
I. Myoclonus.
J. A familiarity problems
K. Hemiballismus
L. Orofacial dyskinesia
Instructions:
For each patient below choose the SINGLE most relevant investigation Options form
the above list. Each Options may be used once, more than once, or not at all.

Q-88 A 15-year-old child is started on Carbamazepine treatment.


Q-89 A young man is started on neuroleptic medication. Within a few days
develops a stiff neck & his trouble closing his mouth.
Q- 90 A man develops difficulty in letting go when he shakes hands.
Q-91 An 8-year-old boy has difficulty getting up for lying position.
Q-92 An elderly man has been on long-term antiemetic therapy
(metoclopramide). His wife complains he cannot stop chewing.

Jen & Adil

Medicbyte
THEME Personality Disorders:
Options:
A. Hysterical personality
B. Schizoid personality.
C. Obsessional personality
D. Sociopthic personality
E. Affective personality
F. Manic personality.
G. Suicidal personality.
H. Immoral personality
I. Asthenic personality
J. Passive dependant personality
K. Depressive personality
L. Sensitive personality
Instructions:
For each patient below choose the SINGLE most relevant investigation Options form
the above list. Each Options may be used once, more than once, or not at all.

Q-93. A 35-year-old lady has a persistent abnormality of mood continually


alternating between euphoria & depression.
Q-94. 36-year-old gentleman is a perfectionist to a rigid degree.
Q-95. A 45-year-old serial killer shows no remorse for crimes he has
committed
Q-96. A 24-year-old lady had has a number of failed relationship & has a
tendency to dramatize limb pains.
Q-97. A shy 27-year-old man is shy of advances has no friends & tends to
be aloof.

Jen & Adil

Medicbyte
THEME Management Of Psychiatric State:
Options:
A. Lithium Salts.
B. Electro Convulsive Therapy.
C. Cognitive Psychotherapy.
D. Group Psychotherapy.
E. Anti-psychotic medication
F. Anti-depressive medication
G. Emergency Care Order.
H. Benzodiazepine medication.
I. Hypnotherapy
J. Behavioral psychotherapy.
K. Bereavement Counseling
L. Chlorpromazine treatment.
Instructions:
For each patient below choose the SINGLE most relevant investigation Options form
the above list. Each Option may be used once, more than once, or not at all.

Q-98. A 56-year-old women says that she sees her deceased husband in &
around the house
Q-99. A 19-year-old female university student undertakes hand washing up
to 30 times in one day. This is getting in the way of her degree work.
Q-100. A 50-year-old vagrant, thought to be alcoholic, is admitted to the
ward where he is being unruly & thinks the nurses are going to kill him.
Q-101. A 24-year-old man says that aliens are taking to him & controlling
his thoughts.
Q-102. A 40-year-old woman is scared of flying.

Jen & Adil

Medicbyte
THEME Diagnosis Of Weight Loss:
Options:
A. Anorexia Nervosa.
B. Carcinoid syndrome.
C. Tuberculosis.
D. Acute Lymphoblastic Leukaemia.
E. Hyperurieaemic Gout
F. Peptic Uleeration
G. Glueagonoma
H. Epstien Barr Virus Infection.
I. Hypothyroidism
J. Cushings syndrome
K. Addisons Mellitus
L. Diabetes Mellitus

Instructions:
For each patient below choose the SINGLE most relevant investigation Options form
the above list. Each Option may be used once, more than once, or not at all.

Q-103. A tall, active lean, 29 year-old single woman is amenorrheaic & has
had recent weight loss.
Q-104. A 26-year-old man has splenomegaly & nose bleeds. His serum uric
acid levels are elevated.
Q-105. A 35-year-old man has headaches. Lymphocytosis & low glucose is
noted on in her cerebrospinal fluid.
Q-106. A 65-year-old woman complains of tiredness. She has developed
dark spots inside her mouth. Hyperkalaemia is noted in her serum
electrolyte profile.
Q-107. A 67-year-old man has hypoglycemia that is very difficult to treat.

Jen & Adil

Medicbyte
THEME: Maternal factors causing neonatal manifestations:
Options:
A. Diabetes.
B. Alcohol.
C. Phenytoin
D. Phenobarbitone.
E. Frusemide.
F. Warfarin
G. Low maternal serum calcium
H. Maternal Mytonia
I. Smoking
J. Intravenous drug abuse.
K. Thalidomide
L. Folate deficiency.
Instructions:
For each patient below choose the SINGLE most relevant investigation Options form
the above list. Each Option may be used once, more than once, or not at all.

Q-108 Microcephaly.
Q-109 Cleft Palate
Q-110 Neonatal Fits
Q-111 Macrosomia.
Q-112 Floppy Infant.

Jen & Adil

Medicbyte
THEME Investigation of Fatigue:
Options:
A. Thyroid Function tests
B. Echocardiogram
C. Electrocardiogram
D. Mono-spot test
E. Serum Urea & Electrolytes
F. Serum Creatinine Kinase levels
G.Digoxin levels
H. Bruce Tread Mill
I. Bacterial Throat swab
J. CXR
K. Urine glucose
L. Hemoglobin
Instructions
For each patient below choose the SINGLE most relevant investigation from the
above list of options. Each Option may be used once, more than once, or not at all.

Q-113 An 8-year-old boy has difficulty getting up from the floor. Mum says
that he does not run around the playground like other children
because he gets tired.
Q-114 A 35-year-old man presents with depression tiredness & weight gain.
Q-115 A 17-year-old girl has enlarged tonsils (white exudates noted) &
tiredness.
Q-116 A 28-year-old intravenous drug abuser develops fatigue petechial
rash & clubbing.
Q-117 A 17-year-old boy complains of tiredness & passing lots of urine.

Jen & Adil

Medicbyte
THEME: Vitamin Deficiency:
Options:
A. Vitamin B12
B. Vitamin B6
C. Vitamin C
D. Vitamin A
E. Vitamin E
F. Vitamin K
G. Vitamin D
H. Folic Acid
I. Biotin
J. Pantothenic acid
K. Niacin
L. Thiamine
Instructions:
For each patient below, choose the SIGNLE deficient vitamin from the above list of
options. Each Option may be used once, more than once or not at all.

Q-118 Peripheral neuropathy, confusion & glossits


Q-119 Associated with neural tube defects
Q-120 Bone X-ray changes white lines noted on long bones of a 5-year-old
boy
Q-121 Creatinuria & muscle weakness
Q-122 Hallucinations & hyperaesthesia
Q-123 Confusion, memory loss & confabulation

Jen & Adil

Medicbyte
THEME: Gynecological Diagnoses:
Options:
A. Candida Vaginitis
B. Gardnerella vaginalis
C. Pelvic Inflammatory disease
D. Hypergonadotrophic amenorrhoea

E. Androgen insensitivity
F. Hypogonadotrophic amenorrhoea
G. Endometriosis
H. Leukoplakia
I. Scabies
J. Lichen sclerosis
K. Human papilloma virus infection
L. Bartholins Cyst
Instructions:
For each patient below, choose the SINGLE most likely diagnosis from the above list
of options. Each Option may be used once, more than once, or not at all.

Q-124 A 14-year-old girl starts having intercourse at the age of 14


Q-125 A 35-year-old women has a thick creamy discharge with vulvar
pruritis
Q-126 A 16-year-old girl has Turners syndrome
Q-127 A 25-year-old girl is a marathon runner for the British Olympic team.
Q-128 An 18-year-old girl with weight loss & distorted body image.

Jen & Adil

Medicbyte
THEME Pelvic Inflammatory Disease
Options:A. Staphylococcus Aureus.
B. Neisseria Gonorrhea
C. Chlamydia trachomatis
D. Escherichia Coli
E. Actinomyces Israeli
F. Myobacterium Tuberculosis
G. Streptococcus Viridians
H. Herpes Simplex
I. Human Papilloma Virus
J. Gardnerella Vaginalis
K. Candida Albicans
L. Sarcoptes scabii
Instruction
For each patient below, choose the SINGLE most likely diagnosis from the above list
of options. Each Option may be used once, more than once, or not at all.

Q-129. A 24-year-old sexually active girl has lower abdominal pain &
pustules on her hands & feet.
Q-130. A 42-year-old lady recently had a tubo-ovarian abscess removed.
She develops pelvic.
Q-131. 30 year odd lady who has an intra uterine device presents lower
abdominal pain.
Q-132.
25-year-old women has very mid lower
Laporoscopy reveals a sever inflammatory process.

abdominal

pain.

Q-133. teenage girl who has just recently started menstruating is admitted
with septic shock & a rash.

Jen & Adil

Medicbyte
THEME Diagnosis Of Neonatal Jaundice
Options:
A. ABO incompatibility
B. Breast Milk Jaundice
C. Group B Streptococcal infection
D. Rhesus disease of the Newborn
E. Hemorrhagic disease of the newborn

F.
G.
H.
I.
J.
K.
L.

Extra hepatic Biliary Atresia


Glucose-6-Phosphate Dehydrogenase deficiency
Sickle cell disease
E. Coli sepsis
Galactosaemia
Gilberts syndrome
Crigler Najer syndrome

Instruction
For each patient below, choose the SINGLE most likely diagnosis from the above list
of options. Each Option may be used once, more than once, or not at all.

Q-134- Mother has blood group A Rhesus Positive her baby is Blood B
Rhesus negative
Q-135- Baby is jaundiced at 12 day with pate stools & urobilogenuria
Q-136- Baby develops increased breathing difficulty by 10 hrs of age & low
blood pressure
Q-137- Neonate is severely jaundice with reducing substances noted in the
urine by dip stick
Q-138- Neonate is re-admitted to hospital with pyrexia & urine positive
dipstick for leucocytes & nitrites

Jen & Adil

Medicbyte
THEME Antimicrobial Prophylaxis:
Options:
A- Penicillin V
B- Pyrazinamide
C- Chloramphenicol
D- Septrin
E- Trimethroprim
F- Rifampicin
G- Erythromycin
H- Ethambutol
I- Isoniazid
J- Antiviral therapy
K- Amoxycillin & Gentamycin
L- Chloroquine
Instruction
For each patient below, choose the SINGLE most likely diagnosis from the above list
of options. Each Options may be used once, more than once, or not at all.

Q-139. An 8-year-old boy with recent onset of peripheral edema & +++
proteinuria
Q-140. A 5-year-old girl with sickle cell disease
Q-141. A 12-year-old boy with impaired T- cell immunity
Q-142. A seven-month old girl who has had previous urinary tract infection
Q-143. Friends of a girl with meningococcal meningitis
Q-144. A 60-year woman with a prosthetic valve who is due for a
gynecological procedure.

Jen & Adil

Medicbyte
THEME Causes Of Haematuria:
Options:
A. Post renal transplant
B. Alports syndrome
C. Group B-streptococcal glomerular nephritis
D. Systemic Lupus Erythematosus
E. E. coli Infection
F. Subacute bacterial endocarditis
G. Nephroblastoma
H. Neuroblastoma
I. Ureteric stones
J. Polycystic kidney disease
K. IgA nephropathy
L. Bladder carcinoma
Instructions:
For each patient below, choose the SINGLE most likely diagnosis form the above list
of options. Each Options may be used once, more than once, or not at all.

Q-145. A 12-year-old boy has sensineural deafness, which is hereditary.


Q-146. A 25-year-old lady presents with apthous ulcers & a non-specific
rash that was brought on after sun bathing.
Q-147. An 8-year-old girl presents with pallor, tiredness. Haematuria is
noted. Full blood count shows anaemia & thrombocytopenia &
electrolytes show acute renal failure.
Q-148. A four-year-old with gross abdominal distension Abdominal palpation
shows a mass in the right flank. There is no family history.
Q-149. A two-year-old is pyrexial, no obvious focus is seen. He is not
irritable.

Jen & Adil

Medicbyte
THEME Investigation Of Arthritis:
Options:
A. Blood cultures
B. C-reactive Protein
C. X-ray of involved joint
D. Bone scan
E. Renal ultrasound scan
F. Echocardiogram
G. Titers for Borrelia Borgderferia Infection

H.
I.
J.
K.
L.

Stools for Bacterial Culture


Stools for Viral Culture
ESR
Serum Uric acid levels
Autoantibody screen

Instructions:
For each patient below, choose the SINGLE most relevant investigation from the
above list of options. Each Options may be used once, more than once, or not at all.

Q-150. A 25-year-old man is admitted complaining of severe knee pain. He


went on a forest walk one week ago
Q-151. A 25-year-old lady has a two-day history of diarrhea. She now
presents with pain on passing urine & conjunctivitis.
Q-152. A 20-year-old had a sore throat three weeks ago. Now he has
developed pain & swelling of his left elbow.
Q-153. A forty-year-old businessman has a acutely tender right hallux.
Q-154. A 20-year-old man develops slow onset back pain, his left heel is
painful to move & he feels he is not a flexible as he used to be.

Jen & Adil

Medicbyte
THEME Investigation Haemolytic Anaemia:
Options:
A. Reticulocyte Count
B. Full blood count
C. Hams test
D. Mono-spot test
E. Coombs test
F. Hemoglobin Electrophoresis
G. Red cell fragility testing
H. Urine Microscopy
I. Thick & Thin blood film
J. Giemsa stain of blood film
K. Echocardiography
L. Glucose-6-Phosphate Dehydrogaenase activity
Instructions
For each patient below, choose the SINGLE most relevant investigation from the
above list of options. Each Options may be used once, ore than once, or not at all.

Q-155. A 12-year-old girl presents with tiredness, splenomegaly & a sore


throat with white exudates.
Q-156. A rhesus positive baby (group A) is born to a mother who is rhesus
negative (group O)
Q-157. A 25-year-old man has mild anaemia (hemoglobin 9-g/dI),
splenomegaly & has had gallstones. He is of Caucasian origin.
Q-158. A 30-year-old has pancytopenia abdominal pain & has had
thrombotic events five years ago.
Q-159. A 30-year-old man returns from holiday to Africa. He is pale on
admission & pyrexial.

Jen & Adil

Medicbyte
THEME Management Psychiatric Illness:
Options:
A. Inform the police
B. Inform special services
C. Ask the parents to exert more control
D. Refer to a dietician
E. Refer to a psychiatrist
F. Refer to the general medical team
G. Reassure
H. Detain under the Mental Health act
I. Sedate
J. Lock in a room
K. Enforce NG feeding immediately
L. Reprimand
Instructions:
Assuming first line management has stabilized each patient described, choose the SINGLE
most relevant next step in management from the above list of options. Each Option may be
used once, more than once, or not at all.

Q-160 A mother brings her child to you with chronic diarrhea. You have
strong grounds to suspect a fractious illness because the nursing staff
have found a number of packets of laxatives in her bag.
Q-161 A 25-year-old lady arrives at the local accident & emergency
department after ingesting an unknown amount of an unknown drug.
Q-162 A patient taking opiates for back pain develops visual hallucinations &
thinks he is God. He recovers & is concerned he has schizophrenia.
Q-163 A young woman develops bradycardia & hypotension. She is very
active & has fine hair on her back.
Q-164 A pt in hospital is assessed to have serious suicidal tendencies. He
wishes to discharge himself.

Jen & Adil

Medicbyte
THEME Management Antepartum Hemorrhage:

Options
A. Fetal Scalp electrode monitoring
B. Full Blood Count
C. Emergency Caesarean section
D. Caesarean section at a late time
E. Ultrasound imaging
F. Blood transfusion
G. CTG monitoring
H. Kleigher test
I. Doppler Ultrasound for fetal heart rate
Instruction
For each patient below, choose the SINGLE most relevant investigation from the
above list of options. Each Option may be caused once, more than once, or not at
all.

Q-165 A 35-year-old lady at 28 weeks gestation has painless vaginal


bleeding. On examination the fetal heart rate is 40 the bleeding setting
soon the admission of the mother.
Q-166 A 20-year-old lady para 3+2, at 32 weeks gestation, is admitted with
sudden onset of abdominal pain. Her blood pressure is 170/105 & heart
rate 105 per minute. Uterine tone is increased with tenderness of
palpation. Feat heart rate is 130.
Q-167 A 29-year-old lady para 3+2, now at 36 weeks gestation has vaginal
bleeding with pain. The uterus does not feet unduly hypertonic on
palpation.

Jen & Adil

Medicbyte
THEME Diagnosis Gastrointestinal Conditions
Options:
A- Jejunal biopsy
B- Antireticulin & antigliadin antibodies
C- Serum Tumor Necrosis Factor Levels

D- ESR
E- Anti DNA stranded antibodies
F- Barium meal & follow through
G- Barium enema & Colonic biopsy
H- Stool for reducing substances
I- Serum electrolytes
J- Serum amylase
K- Arterial Blood gas
L- Lateral & AP Abdominal X-ray
Instructions
For each patient below, choose the SINGLE most relevant investigation from the
above list of options. Each Option may be used once, more than once, or not at all.

Q-168 A 15-year-old lady has had tiredness for the last four years. She
has delved puberty & has been told she shouldnt eat gluten-containing
foods.
Q-169 A 25-year-old man has diarrhea on & off. On examination he has
finger clubbing.
Q-170 A four-year-old girl has had gastroenteritis recently. Her mother
complains that her diarrhea is still continuing 2 weeks after the illness
began.
Q-171 A 25-year-old man who was driving was involved with a high-speed
collision. He was wearing his seat belt & now complains of upper
abdominal pain.
Q-172 A 80 years old lady has bilious vomiting & abdominal pain. On
examination tenderness & abdominal distension is noted.

Jen & Adil

Medicbyte
THEME: Recognition ECG Abnormalities
Options:
A- angina
B- Hypertension
C- Digoxin toxicity
D- Severe pneumonia
E- Hypokalaemia
F- Hypothyroidism
G- Hypothermia
H- Hypocalcaemia
I- Hypercalcaemia
J- Hyperkalaemia
K- Acute myocardial Infarction
L- Pulmonary Embolism
Instructions
For each description below, choose the SINGLE most likely condition responsible
from the above list of options. Each Option mar be used once, more than once or
not a all.

Q-173 Atrial Fibrillation & S1 Q3 T3 pattern (Deep S wave in lead I,


pathological Q wave in III & inverted T waves in III)
Q-174 Widespread ST depression greater than 2 mm on treadmill testing.
Q-175 Reverse tick sign (down sloping ST depression in V5 & V6)
Supraventricular tachycardia & first-degree heart block.
Q-176 5mm ST elevation in all anterolateral leads (I, II, avL, & V1-V6)
associated with early T wave inversion.
Q-177 Tall tended T waves, wide QRS complex, absent P wave.

Jen & Adil

Medicbyte
THEME: Recognition Of Abnormalities
Options:
A Pulmonary Embolism
B Angina
C Hyperkalaemia
D Hypokalaemia
E Hypothyroidism
F Hypothermia
G Pericardial effusion
H Hypercalcaemia
I Hypertension with LVH
J Acute pericarditis
K pneumonia
L Pericardial Effusion
Instructions:
For each description below, choose the SINGLE most likely condition responsible
from the above list of Options may be used once, more than once, or not at all.

Q-178 J-waves, bradycardia & first-degree heart block


Q-179 Prolonged QT interval, all ST segments are elevated (except in avR)
& Show characteristic saddle shape (concavity upwards)
Q- 180 Globally small complexes (ECG machine is working properly)
Q-181 Tall QRS complexes (sum of the S wave in V-I & the R wave in V6 >
35 mm) with left ventricular strain pattern.
Q-182 Atrial Fibrillation, self-resolving.

Jen & Adil

Medicbyte
THEME: Management Of Headache
Options:
A. Listen, counsel & reassure
B. ESR, High dose steroids
C. Repeated Lumbar puncture, dexamethasone
D. CT head scan & lumbar puncture
E. History & clinical features
F. Electroencephalography
G. Skull XR
H. Autoantibody screen
I. Sinus XR
J. Visual acuity
K. High dose Intravenous antibiotics
L. ECG.

Q-183 A 52-year-old man presents with chronic dull & like headache with scalp
tenderness, worse at night.
Q-184 A 17-year-old girl as admitted with headache & seizures. O/E she has
papilloedema, bradycardia & a petechial rash.
Q-185 An 18-year-old girl presents with premenstrual
associated with vomiting & a need to he in the dark.

unilateral

headache

Q-186 A 58-year-old woman with pain in her jaw on eating headache & fleeting
visual disturbances.
Q-187 A 21-year-old student described severe headache (like being hit over the
back of the head) followed by vomiting lift sided weakness & loss of
consciousness. Initial fundoscopy is normal.
Q-188 A 32-year-old obese woman complains of headache on walking which is
worsened by coughing. She also has blurred vision & occasionally sees double.
CT head scan has ruled out a space occupying lesion. She feels entirely well
otherwise & there is no impairment of consciousness.
Q-189. A teenaged describes constant unilateral frontal pain with local tenderness

Jen & Adil

Medicbyte
THEME Recognition causes of meningitis:
Options:
A. Malaria
B. Leptospirosis
C. Paramyxovirus
D. Haemophilus Influenza
E. Neisseria meningitides
F. Streptococcus Aureus
G. Staphylococcus Aureus
H. Mycobacterium tuberculosis
I. Treponema pallidum
J. Epstein Barr Virus
K. Candida Albicans
L. Cryptococcus neoformans
Instructions:
For each description below, choose the SINGLE most likely condition responsible from the
above list of Options may be used once, more than once, or not at all.

Q-190 Sewer worker presents with jaundice, fever, & signs of meningism,
painful calves & brushing.
Q-191
A teenager develops a positive Kernings sign & a rash that
doesnt blanch on pressure.
Q-192
An Intravenous drug abuser with established AIDS presents
with suspected meningitis. The organism is confirmed in the CSF by
India Ink Stain.
Q-193
An 18-year-old boy presents with severe unilateral pain &
swelling over the parotid area. He subsequently develops headache,
neck stiffness & photophobia.
Q-194
A 45-year-old man returns from holiday & develops a dry
cough. His wife complaints that the sweats excessively at night. When
he finally coughs up blood, she takes him to the GP where he also
admits to chronic vague headache, loss of appetite, intermittent
vomiting & an episode of collapse at work during which he was seen to
jerk his limbs.

Jen & Adil

Medicbyte
THEME Common drug problems
Options:
A. Atenolol
B. Gentamycin
C. Metoclopramide
D. Metronidazole
E. Ampicillin
F. Lithium
G. Digoxin
H. Rifampicin
I. Captopril
J. Phenytoin
K. Verapamil
L. Warfarin
Instructions
For each description below, choose the SINGLE most likely condition responsible
from the above list of Options may be used once, more than once, or not at all.

Q-195
A patient with mild asthma (not on treatment) is treated for
angina. His asthma subsequently deteriorates.
Q-196
A patient with angina on maximal therapy develops severe
constipation
Q-197
A 54-year-old man has a myocardial infraction & is treated
according to guidelines. He is discharged home & is lost to follow up. 6
months later, he is admitted with a diagnosis of renal failure.
Q-198
A 22-year-old lady on treatment for a chronic disease develops
ataxia, nystagmus, dysarthria & diplopia. She is noted to have gum
hyperplasia.
Q-199
A man on treatment for a heart valve defect has an alcohol
binge & vomits blood.
Q-200
A girl with sore throat, palatal exudates, lymphadenopathy &
splenomegaly develops a widespread rash.

Jen & Adil

Medicbyte
THEME Important Drug Interactions
Options:
A. Oral contraception & erythromycin
B. Warfarin & Rifampicin
C. Cyclosporin A & Phenytoin
D. Captopril & K+ sparing diuretic
E. Propranolol & Verapamil
F. Theophylline & Erythromycin
G. Lithium & bendrofluazide
H. Digoxin & Frusemide
I. Warfarin & cimetidine
J. Metformin & cimetidine
K. Phenytoin & Isoniazid
L. Penicillamine & tetracycline.
Instructions
For each description below, choose the SINGLE most likely drug responsible from
the list of Options may be used once, more than once, or not at all.
Q-201 A elderly man on treatment for an irregular heart rate develops ankle
swelling for which he is given a new drug by his doctor. 2 weeks later he
develops complete heart block, nausea & c/o seeing yellow.
Q-202 A patient on treatment for a mood disorder is noted to have hypertension for
which she is prescribed a new drug. Within a week she develops tremor,
agitation & 4 weeks later presents with heart block, seizures & a raised
creatinine of 400.
Q-203 A young career woman is treated by her GP for a resp: infection. A week
later, she develops irregular bleeding 6 week after this, pregnancy test is +ve
Q-204 A man develops a persistent arrhythmia for which DC cardioversion is
planned. He is commenced on appropriate treatment for the 4 weeks leading
up cardioversion. He plans a holiday during which he contracts tuberculosis.
On week after starting treatment for this, he develops sudden onset left sided
weakness.
Q-205 A young woman is maximal therapy for rheumatoid arthritis. She is also on
treatments for acne. At routine follow up, her urea is 30 & creatinine 600.

Jen & Adil

Medicbyte
THEME Monitoring Of Drug Side Effects
Options:
A. Thyroid function tests
B. Pulmonary Function Tests
C. Urea & Electrolytes
D. INR
E. Blood Levels
F. CXR
G. ECG
H. APTT
I. Blood Glucose
J. Arterial Blood Gases
K. Urinalysis
L. Liver function tests.
Instruction:
For each description below, choose the SINGLE most likely condition responsible
from the above list of Options may be used once, more than once, or not at all

Q-206

Amiodarone

Q-207

Captopril

Q-208

Theophylline

Q-209

Frusemide

Q-210

Gentamyein

Jen & Adil

Medicbyte
THEME Investigation of altered consciousness / blackouts
Options
A. CT head scan
B. Lying & standing blood pressure
C. Echocardiography
D. 24 hour electroencephalography
E. Arterial Blood Gases
F. Lumbar Puncture
G. ECG during carotid sinus massage

H.
I.
J.
K.

CXR
Urinalysis
Blood glucose
Nerve conduction studies

Q-211 An 18-year-old girl develops altered breathing & aphasia while


watching TV. She is seen to pick at her clothes.
Q-212 A 62-year-old man on treatment for heart failure loses consciousness
on standing.
Q-213 A 32-year-old man presents with intermittent blackouts, mid systolic
murmur & ejection click on auscultation.
Q-214 A young woman presents with episodes of generalized paraesthesia,
stiffening of the limbs, loss of urinary continence, tetany & an inability
to gain breath. She says she remembers nothing of the event.
Q-215 A 60-year-old man presents with headache (worse on coughing),
vomiting & blurred vision.
Q-216 An elderly man loses consciousness every time he turns his head to
the left.
Q-217 A young man has an episode of shaking, sweating & hunger before
losing consciousness. He then has a seizure.

Jen & Adil

Medicbyte
THEME: Causes Of Confusion:
Options:
A. Infection Screen
B. Mental State Questionnaire
C. CXR
D. ECG
E. Arterial Blood gases
F. Red cell transketolase
G. Blood glucose
H. Rental Function
I. History
J. ECG
K. CT head scan
L. Lumbar puncture
Instruction
For each patient below, choose the SINGLE most relevant investigation from the
above list of options. Each Option may be used once, more than once, or not at all.

Q-218 A 78-year-old woman presents with slow onset forgetfulness, loss of


abstract though, dysphasia, emotional blunting & visual hallucinations.
She has no family or friends.
Q-219 A 48-year-old
nystagmus.

homeless

man

develops

ataxia,

confusion

&

Q-220 A 62-year-old man on treatment for heart failure (Frusemide),


arrhythmia (Digoxin) & again (enalopril) presents with confusion &
pallor.
Q-221 A 73-year-old woman bed-bond after an extensive stroke develops
fluctuating confusion & fever.
Q-222 A 24-year-old man collapses at a bus stop & passes urine.

Jen & Adil

Medicbyte
THEME Investigation Of Anaemia:
Options:
A. Thrombin Time, Fibrin
degradation products
B. Rectal examination
C.

Hemoglobin
electrophoresis

D. Sickle test
E.

Urine for Bence Jones


protein. Bone X-Ray

F. Hams test
G. Echocardiogray
H. Coombs test
I. Iron screen (total Serum
Iron, ferritin, total iron
binding capacity)
J. Pyruvate Kinase assay
K. Serum Folate
L. Serum B12
Instruction
For each patient below, choose the SINGLE most relevant investigation from the
above list of options. Each Options may be used once, more than once, or not at all.

Q-223 A 62-year-old woman presents


paraesthesia & a sore red tongue.

with

tiredness,

dyspnoea,

Q-224 A 19-year-old girl presents with mennorrhagia & anaemia.


Q-225 A 25-year-old woman develops uncontrolled bleeding post caesarian
section.
Q-226 A 60-year-old man presents with weight loss, back & shoulder pain,
hypercalcaemia & renal failure.
Q-227 A 16-year-old Asian girl presents with anaemia. She remembers her
brother died at the age of 5 after an illness since birth, which required
repeated transfusion.

Jen & Adil

Medicbyte
THEME Investigation of endocrine disease:
Option:
A. Dexamethasone suppression test
B. ACTH stimulation test
C. Fasting blood glucose x 2
D. Serum aldosterione
E. Urinary ketones
F. T3, T4, TSH, Microsomal antibodies

G.
H.
I.
J.
K.
L.

HB AIC
Look at old photograph
Basal plasma prolactin
Water deprivation test
Liver function tests
C peptide

Instruction:
For each patient below, choose the SINGLE most relevant investigation from the
above list of options. Each Options may be used once, more than once, or not at all.

Q-228 An 12-year-old boy presents with 4 Weeks of weight loss, polyuria &
polydispsia
Q-229 A 44-year-old women presents with tachycardia, Atrial fibrillation,
double vision & swelling above her ankles. She has lid lag on
examination.
Q-230 A 42-year-old man has hypertension, hyperglycemia, myopathy,
thinning of the skin, buffalo hump & truncal obesity.
Q-231 A 34-year-old man presents with insidious onset weakness & weight
loss. O/E, he has hyperpigmentation of the palmar creases & postural
hypotension.
Q-232 A 48-year-old man is admitted for investigation of hypertension &
glycosuria. His wife comments that his appearance has changed over
the last few years & everything seems to have got bigger. He also
complains of tingling in left hand & excessive sweating.

Jen & Adil

Medicbyte
THEME: Management of acute poisoning
Options:
A Dimercaprol
B Atropine
C Flumazenil
DDicobalt edatate
E Specific antibody fragments
F Antitoxin
GDesferryoxamine
HNaloxone
I Phytomenadione
J Fresh frozen plasma & Vitamin K
K N acetyleysteine
L Protamine Sulphate
Q-233 A 35-year-old lady has taken an entire bottle of beta-blockers
tablets.
Q-234 A 15-year-old boy has taken oral benzodiazepines, & now has
difficulty breathing.
Q-235 A 17-year-old girl has taken her mothers Warfarin. She is not
actively bleeding.
Q-236 A 35-year-old man has taken dioxin in over dosage.
Q-237 A 24-yr man has overdosed accidentally on painkillers & is admitted
with breathing difficulty & pinpoint pupils.
Q-238 A 6-year-old girl has taken all he mother iron tablets
Q-239 A 15-year-old girl is rushed to A&E after heaving taken 30
Paracetamol tablets.
Q-240 A 20-year-old lady is rushed in to hospital after being found
moribund. She bad eaten old tinned meat soup.
Q-241 A 35-year-old lady has cyanide poisoning
Q-242 Warfarin tablets have been taken by a young woman. She is actively
bleeding.

Jen & Adil

Medicbyte
THEME Recognition clinical emergencies:
Options:
A Tension pneumothorax
B Coning
C Disseminated intravascular coagulation
D Meningococcal Meningitis
E Massive Pulmonary embolism
F Malignant hypertension
G Addisonian Crisis
H Diabetic Emergency
I Sickle crisis
J Acute myocardial infarction
K Aortic dissection
L Mismatched blood transfusion

Q-243 A 20-year-old man is admitted with a headache, which he describes


as being hit over the back of the head. 30 minutes later, he has a
respiratory arrest. His left pupil is dilated.
Q-244 A 32 year footballer is admitted with breathlessness. His respiratory
rate is 30, hart rate 180, blood pressure 78/44, his neck veins are
distended. His trachea is not in the middle.
Q-245 22-yr woman with headache, photophobia ad a rash. A few hors
later she beings to bleep profusely from a venepuncture site & rapidly
becomes tachycardia & hypotensive.
Q-246 A 40-year-old man rapidly develops pallor, tachycardia, hypotension.
His urine output falls & he is seen to shake vigorously. He had been
admitted the day before for investigation of a profound anaemia.
Q-247 A 50-year-old woman collapses 10 days after surgery. She is
hypotensive, has a gallop rhythm on auscultation, a pleural rub &
distended neck veins.

Jen & Adil

Medicbyte
THEME Investigation of renal disease:
OPTIONS:
A
Urinary protein
B
Rectal examination
C
Cystoscopy
D
Renal ultrasound
E
Urine microscopy & culture
F
Renal angiography
G
KUB X ray
H
Glomerular Filtration Rate
I
Urinary electrolytes
J
Cyclosporin level
K
CXR
L
Intravenous urography
Instructions:
For each patient below, choose the SINGLE most relevant diagnostic from the above
list of options. Each Option may be used once, more than once, or not at all.

Q-248
A 64-year-old man develops urinary hesitancy & dribbling. He is
frustrated, that he has to wake up to pass urine frequently, especially
since his back has been very sore lately.
Q-249
A 30-year-old woman presents with ankle swelling & frothy
urine. Examination is normal.
Q-250

A 4-year-old girl presents with abdominal pain, dysuria & fever.

Q-251
A 22-year-old man presents with abdominal pain & raised blood
pressure. He complains of blood in his urine. He thinks his father &
brother have the same thing.
Q-252
A 45-year-old is admitted with excruciating left sided
abdominal pain radiating from his back to his groin. He says it is the
worst pain he has ever had.

Jen & Adil

Medicbyte
THEME Dermatological Conditions:
Options:
A. Dermatitis herpetiformis
B. Pompholyx
C. Eczema
D. Psoriasis
E. Erythema nodosum
F. Lichen planus
G. Epidermolysis bullosa
H. Pityriasis Rosea
I. Adenoma Sebacum
J. Hereditary angioderma
K. Erythema Multiform
L. Pemphigus

Q-253
A 25-year-old lady has well demarcated salmon pink lesions
with a silvery scaling mainly of the extensor surfaces of her arms.
Q-254
A 46-year-old lady has flat papules, which are purplish in color
of her wrists & ankles. Theses are itchy. Also a fine white lacy
patterning is noted over the papule.
Q-255
A 25-year-old nurse had a small erythematous patch on her
chest, which disappeared. Following these, seven days later, multiple
patches are noted on her front & back which are brown in color, macular
& discrete. These are mildly pruritic. The lady is very well in herself.
Q-256
A 25-year-old lady has had problems over most his life with
swelling of his limbs associated with purpura. Recently he had such am
episode associated with severe breathlessness.
Q-257
In a 25-year-old man erythematous papules (bullet lesions) are
noted in the back of the hands and on the arms. There is a central area
of Pallor and edema within these papules.

Jen & Adil

Medicbyte
THEME Causes Of Pneumonia
Options:
A. Bacteroides fragilis
B. Coxiella burrnetii
C. Escherichia coli (Gram-ve)
D. Haemophilus influenzae
E. Legionella Pneumophilia
F. Mixed growth of organisms
G. Mycobacterium tuberculosis
H. Mycoplasma Pneumoniae
I. Pneumocystis carinii
J. Staphylococcus Aureus.
K. Streptococcus Pneumoniae

Q-276
A 25-year-old man has a three-day history of shivering, general
malaise & productive cough. The x-ray shows right lower lobe
consolidation.
Q-277
A 26-year-old man presents with severe shortness of breath &
dry cough which he has had for 24 hours. He is very distressed. He has
been an IV drug user. The x-ray shows peri-hilar fine mottling.
Q-278
A 35-year-old previously health man returned from holiday five
days ago. He smokes 10 cigarettes a day. He presents with mild
confusion, a dry cough & marked pyrexia. His chest examination is
normal. The x-ray shows widespread upper zone shadowing.
Q-279
A 20-year-old previously health woman presents with general
malaise, severe cough & breathlessness which has not improved with a
seven day course of Amoxycillin. There is nothing significant to find on
examination. The x-ray shows patchy shadowing through out the lung
fields the blood film shows clumping of red cells with suggestion of cold
agglutinins.

Jen & Adil

Medicbyte
THEME Rx of menopausal symptoms
Options:
A. Clonidine
B. Combined hormone replacement therapy (HRT)
C. Dietary modification
D. Hypnotic preparations
E. Mineral supplements
F. Estrogen only HRT.
G. Psychological support
H. Referral to psychiatrist
I. Regular exercise
J. Vaginal lubricant
K. Vaginal estrogens.
Instruction
For each patient below, choose the SINGLE most relevant diagnostic from the above
list of options. Each Option may be used once, more than once, or not at all.

Q-280. A 56-year-old woman whose periods stopped five year ago has
become increasingly depressed. She now feels life is no longer worth
living & threatens suicide.
Q-281. A 72-year-old woman has experienced frequency of micturition
intermittently for the last a few months. Mild stream urine (MSU)
cultures have been persistently negative. She is well otherwise, but
would like the symptoms resolved.
Q-282. A married 52-year-old woman who has a family history of breast
cancer has been experiencing mild discomfort for a few hours following
intercourse for the last month. She is worried about using hormones.
Q-283. A 45-year-old woman who has had a total abdominal hysterectomy
(TAH) & bilateral salpingo-oophorectomy (BSO) for fibroids &
menorrhagia complains of hot flushes, night sweats & mood swing. She
has no other medical problems.

Jen & Adil

Medicbyte
THEME: Diagnosis Of Headache.
Options:
A. Tension headache
B. Migraine
C. Cluster headache
D. Epilepsy
E. Giant eel arteritis
F. Raised intracranial pressure
Instruction:
For each patient below, choose the SINGLE most relevant diagnostic from the above
list of options. Each Option may be used once, more than once, or not at all.

Q-284. 62-year-old woman complains of headache & jaw claudication during


eating Symptoms.
Q-285. 45-year-old male was referred to neurologist with history of
headache. He says that headache is quite severe, beings at the same
time very day & last about 1 hour. Pain is almost always on the left &
accompanied by lacrimation & nasal congestion. He had similar episode
last year that lasted approximately 2 months.
Q-286. 35-year-old woman, (professional, 2 children, divorced,) complains
of headache, which she describes as a tight band round the head. The
pain is bilateral & presents for more than a year.
Q-287. 28-year-old woman taking contraceptive pills complains of episode
of severe throbbing headache. She noted that before the pain beings she
experiences some funny feelings of numbness & tingling in the right
hand that quickly spread all-over.

Jen & Adil

Medicbyte
THEME Diagnosis Of Sore Throat.
Options
A. Diphtheria
B. Infectious mononucleosis.
C. Acute follicular tonsillitis
D. Scarlet fever
E. HIV
F. Agranulocytosis
G. Acute otitis media.
Instruction
For each patient below, choose the SINGLE most relevant diagnostic from the above
list of options. Each Option may be used once, more than once, or not at all.

Q-288. 5-year-old child complains of sore throat & earache. He is pyrexial.


On examination tonsils enlarged & hyperaemic, exude pus pressed
upon.
Q-289. 25-year-old man presents with sever malaise & sore throat. He has
red tonsils with yellow membranes & enlarged lymph nodes on the
neck.
Q-290. 30-year-old man presents in Accident & Emergency department with
difficulty in breathing. He has returned from India recently. Examination
of his throat revealed gray membranes on the tonsils & uvula. Pyrexia is
mild
Q-291. 7 years old woman presents to her GP with sore throat, a punctate
erythematous rash & circum-oral pallor.
Q-292. 30-year-old woman presents to her GP with store throat. She is
being treated for thyrotoxicosis currently.

Jen & Adil

Medicbyte
THEME Treatment Of Diabetes
Options:
A. Chlorpropamide
B. Insulin
C. Tolbutamide
D. Metformin
E.
F.
G.
Instruction:
For each patient below, choose the SINGLE most relevant diagnostic from the above
list of options. Each Options may be used once, more than once, or not at all.

Q-293.
25-year-old man complains of thirst, polyuria & general ill
health. Investigations revealed hypoglycemia & glucosuria. Past
medical history episode of ketoacidosis.
Q-294. 78-year-old man non-obese was found to have type 2 diabetes
mellitus. He is hypertensive & suffers from ischaemic heart disease.
Q-295. 50-year-old lorry driver was found to have type 2 diabetes mellitus.
He is slightly overweighed & failed to improve o dietary measures
alone.
Q-296. 58-year-old school teacher has past medical history of hyperosmolar
coma. His weight is normal. Dietary measures failed to control his
blood sugar.

Jen & Adil

Medicbyte
THEME Investigation Of Confusion
Options:
A. Blood Cultures
B. Blood glucose
C. Chest X-ray
D. CT scan of head
E. Electrocardiogram (ECG)
F. Full blood count (FBC)
G. Mid-stream specimen of urine
H. Stool culture
I. Thyroid functions tests
J. Ultrasound abdomen
K. Ultrasound abdomen
L. Urea & electrolytes
Q-297. An 84-year-old woman in nursing home has been constipated for a
week. Over the past few days she has become increasingly confused &
incontinent
Q-298. A previously well 78-year-old woman has been noticed by her
daughter to be increasingly slow & forgetful over several months. She
has gained weight & tends to stay indoor with the heating on even warm
weather.
Q-299. A 64-yr man has recently been started on tablets by his GP. He is
brought to the A&E department by his wife with sudden onset of
aggressive behavior, confusion & drowsiness. Prior to starting tablets he
was losing weight & c/o thirst.
Q-300. A frail 85-year-old woman presents with poor mobility & recent
history of falls. She has deteriorated generally over the past two week
with fluctuating confusion. On examination she has a mild right
hemipareis
Q-301. A 75-year-old man with know mild Alzheimers disease became
suddenly more confused yesterday. When seen in the Accident &
emergency department, his BP was 90/60 & his PR was 40/min &
regular.

Jen & Adil

Medicbyte
THEME Diagnosis Muskuloskeletal Pain:
Options:
A. Osteoid Osteoma
B. Chondroma
C. Compact osteoma
D. Ewings sarcoma
E. Haematoma
F. Stress fracture
G. Myositis ossificans
H. Pyogenic infection
I. Disc prolapse
J. Malignant metastasis

K.
L.
M.
N.
O.
P.
Q.
R.
S.

Spondylosis
Malingering
Spondylolisthesis
Arachnoiditis
Scheuermanns disease
Calves Disease
Brodies absees
Overuse injury
Pathological Fracture.

Instruction.
For each patient below, choose the SINGLE most relevant diagnostic from the above
list of options. Each Option may be used once, more than once, or not at all.

Q-302. A 14-year-old girl noted for playing truant from school. Otherwise fit
complains of backache & fatigue. Her parent are suspicious & also
noticed notice that the girl is increasingly becoming round shouldered
on examination movements are normal & a smooth hump in the
thoracic region is seen.
Q-303. Following an injury, a 27-year-old youth gradually developed a
tender swelling near the elbow joint. X rays shows a fluffy density in
the soft tissue near the joint.
Q-304. A 28-year-old youth develops persistent pain in the right lower leg.
The patient is an avid weight lifter & ignores the pain. The youth had
been advised earlier by a physiotherapist to exercise in moderation.
The youth notices some wasting of the affected area a few days later.
On subsequent X ray examination a radiolucent area.
Q-305. A 32-year-old man while lifting weights develops severe back pain &
is unable to straighten up. He is in considerable pain & notices that the
pain is made worse by straining later there is numbness in the left leg.
Q-306. A 45-year-old man with along history of back pain & invasive
investigation complains bitterly of diffuse back pain. He complains of
impotence & cramps & burning of the lower limbs.

Jen & Adil

Medicbyte
THEME
1-D This is small bowel lymphoma which is associated with coeliac
disease. The other carcinoma associated is gastric carcinoma. The
lymphoma characteristically, primarily involves the jejunum.

2-A. This diagnosis should be thought of if there are visual


disturbances
in
a
child
with
stunted
growth.
The
craniopharangioma impinges on the optic chiasm causing
symptoms.

3-K. It is Addisons disease (adrenocortical insufficiency) the most


common cause of which is autoimmune destruction or tuberculosis
infection. Other causes include malignant destruction or amyloid
disease. Postural hypotension can occur.
4-C or D.
This young man has Cushings triad (vomiting,
bradycardia & hypertension) & is due to acute increased
intracranial pressure. Any intracranial tumor may cause this.
Therefore both (C) & (d) are correct . in the examination it is likely
that there will not be more than one correct answer for each stem.
C/F of an intracranial tumor depends on the site of the tumor.
Epilepsy
Increases Intracranial Pressure
False localizing sings
Local effects on neighboring.
Cushings triad: increase in Intracranial Pressure
Cushings reflex: post myocardial infarction.
Cushings disease: due to a central increase in central ACTH
production.
Cushings Syndrome: due to an adrenal increase in Cortisol
5-I
Carcinoid tumour:25% affect the appendix & ileum 5%
metastasize to the liver & produce
the symptoms above
Symptoms without metastasis are rare. Associations: Cardiac
abnormalities (tricuspid incompetence, pulmonary stenosis) and
Cushings Syndrome.

Jen & Adil

Medicbyte
THEME
6-A The middle ear is the port of entry for the Streptococcus
infection.

7-B Children with rental tract anomalies have predisposition to


Urinary tract infections, especially E. coli infection.

8-G

9-C This is tuberculous meningitis

10-L Lymes disease

11-C Pulmonary TB 30-50% of HIV patients in the developed


world develop TB & the figure is rising. Mantoux tests may be
negative due to impaired immunity & presentation may be atypical.
Past BCG vaccination may not confer protection.

Jen & Adil

Medicbyte
THEME
12-A

13-F
14-E

.
15-J

16-K
This is acute immune thrombocytopenia. Treatment is
supportive as the condition usually resolves.

17-L
This is chronic immune thrombocytopenia. Treat with
steroids & consider spleenectomy.

Jen & Adil

Medicbyte
THEME
18-C or B Cerebello-pontine angle tumors. ipsilateral deafness,
nystagmus & reduced reflex. Ipsilateral 5th & 7th never palsyes
(lower motor neuron signs) & cerebellar signs may occur.

19-A
Hepatolenticular degeneration due to excess copper
deposition = Wilsons disease Recessive disorder of copper
metabolism causing basal ganglia (extrapytamidal) & liver signs.
Look for Kaiser Fleischer rings in the eves. High plasma copper with
low caeruloplasmin.

20-E Guillain Barre syndrome Acute postinfectious demyelinating


polyneuritis. Follows days or weeks after mild infection. Progressive
ascending polyneuropathy common. Sensory symptoms occur but
difficult to detect. Be alert to respiratory involvement, intubation
may be required (monitor vital capacity regularly). Cerebrospinal
fluid protein is high

21-F
other causes polyneuropathy. Diabetes Mellitus, renal
Failure, Liver failure, alcoholism hypothyroidism, malignancy,
vasculitis, deficiency, lead, Metronidazole & Phenytoin

22-C Polio 7 days incubation then 2 days flu like prodrome. Preparalysis stage (fever tachycardia, headache, vomiting, neck
stillness) proceeds to paralysis stage(lower motor neuron sign due
to anterior horn cell damage, myalgia, respiratory failure). Very
rare in UK.

Jen & Adil

Medicbyte
Theme:
Note that a blood film cannot be megaloblastic this refers to marrow
appearances. The corresponding blood film picture is Macrocytosis

23-A A variety of hold films may be seen in Coeliac disease. In the


above case, Macrocytosis occurs due to floate/B12 malabsorption.
Iron can also be malabsorbed & there may be a mixed
macrocytic/microcytic picture. Splenic atrophy is a rare association
in which case Howell Jolly bodies (hypersegmented neutrophils)
may be a seen on a blood film. Lastly anaemia with a normal blood
film(anaemia of chronic disease) may be seen.
24-C
25B Lead toxicity can cause microcytosis on a blood film. Other
are basophilic stippling & pappenheimer bodies
26-E Howell Jolly bodies occur in spleenectomy (in this case
autospleenectomy from recurrent splenic infarction). They are
described above
27-G
Mveloma with Leucoerythroblastic blood film(immature
nucleated
red &
white
cells)
seen
in bone
marrow
infiltration(metastases, mveloma, chronic myeloid leukemia
lymphoma.
TB)
or
bone
marrow
stress
(sever
haemorrhage,trauma,septicaemia,hypoxia).
28-H
Microangiopathic Haemolytic anaemia (helmet cells,
fragmented red cells, polychromasia, reduced platelets &
spherocytes on film) is seen in DIC (this case), thrombotic
thrombocytopeniac purpura, malignant hypertension, gram
negative sepsis & obstetric complications. Clotting factors are
consumed. Uncontrolled bleeding occurs & fibrin strands form a
mesh in blood vessels which fragments cells.
29-L Atypical lymphocytes are seen on Infectious mononucleosis
(this case). They consist of T cells reacting to infected B cells Also
seen in CMV infection, flu, toxoplasmosis & hepatitis A infection.
30-C Dimorphic blood picture = macrocytes & microcytes. Due to
mixed deficiencies although also seen in sideroblastic anaemia,
treated iron deficiency & recent blood transfusion.
31-K
Parasite infection (Ascaris, Strongyloides, Anylostoma,
Filiariasis, Schistosomiasis) causes Eosinophilia in the blood film.
Also seen in fungal infection, drugs (non steroidal, sulphonamides),
vasculitis (PAN, sarcoid & Churg Strauss syndrome.
32-B
Iron deficiency anaemia blood films in hypochromic &
microcytic with target cells, pencil cells & poikilocytosis.
33-D Anaemia of chronic disease.

Jen & Adil

Medicbyte
THEME:
34-A

35-B Dialysis may be required in the treatment of HUS for renal.


Adults have worse prognosis than children (80% may require
chronic dialysis). Blood him shows microngiopathic haemolysis

36-C

37-H Purple spots that do not blanch with pressure intradermal


bleeding (purpura). IN HSP this occurs mainly on the extensor
surfaces of the legs & buttocks. 1/3 have nephritis. Arthritis &
abdominal pain complete the picture. Often follows upper
respiratory tract infection (URTT) & has a benign prognosis.

38-J Most common glomerulonephritis in the developed word.


Usually affecting young men after a respiratory infection. IgA
deposits in glomeruli. Heavy proteinuria & hypertension indicate a
poor prognosis (15% develop chronic renal failure). Can occur
secondary to Ankylosing spondylitis HIV & Coeliac disease.

Jen & Adil

Medicbyte
THEME:
39-C Low CO2 & normal/elevated O2 might be sign on arterial
gases form someone who is hyperventilating. Hyperventilation may
be hysterical (this case), or secondary to central neurological
disease (stroke, sub arachnoid hemorrhage, meningitis)

40-A

41-B Metabolic acidosis occurs due to ingestion of acid (Salicylic).


The patient will soon begin to hyperventilate as the body tries to
compensate by exhaling CO2. Metabolic acidosis can also be seen
with increase in lactic acid (exercise shock, hypoxia, liver tailure,
biguanide therapy, trauma), metabolic disease (diabetes, alcoholic
poisoning, starvation) & uremia, addisons disease, renal tubular
disease & diarrhea.

42-B

43-A occurs secondary to K+ depletion

Jen & Adil

Medicbyte
THEME:
44-H Brushings, washings & biopsy of bronchial lesions provides
rapid, histological diagnosis of bronchial carcinoma. If neck nodes
present, do fine needle aspiration first. May obviate the need for
bronchoscopy.

45-B Farmers lung, (hypersensitivity reaction to Mircopolyspera


Faeni spores in hay) Symptoms initially occur 4-8 hours after each
exposure & may become unremitting with repeated exposure.

46-J
Massive pulmonary embolism constitutes a clinical
emergency. Management varies according to local facilities &
certainty of diagnosis. With this classical picture it would be wise to
consider immediate embolectomy. If not compromised, treat with
oxygen, analgesia, iv heparin & organize a V/Q scan, spiral CT or
pulmonary angiogram depending on protocol & availability. Gases
would be important but do not confirm the diagnosis.

47-E
Sarcoidosis. Glaucomatous multi system disease. First
presentation Mostly chest. Diagnosis is based on clinical features &
Demonstration of non causing granuloma on histology. Granulomas
Produce. Angiotensin converting enzyme (increases calcium)

48-C Tuberculosis, there is a high ethnic population in Glasgow.

Jen & Adil

Medicbyte
THEME:
This Question is designed to test understanding of obstructive & restrictive
respiratory disease.
Obstructive defect (asthma, emphysema, bronchitis, tumor ) = FEVI /
FVC ratio < 70%. Asthma shows reversibility to inhaled bronchodilators &
diurnal variation, emphysema usually doesnt although mixed pictures can
exist.
Restrictive defect. (interstitial lung disease, neuromuscular disease,
infiltrative disease, chest wall disease such as kyphoscolisosis, effusions,
obesity) = FEVI / FVC ratio > 70% A restrictive defect should be
confirmed by measuring the total lung capacity ( TLC: reduced in
restriction). If neuromuscular disease is suspected, measure inspiratory &
expiratory mouth pressures.

49-J Evidence of reversible airways obstruction is the hallmark of


asthma. FEVI / FVC ratio 60-80% = mild obstruction, 40-60% =
moderate, < 40% = severe
Transfer factor can be normal, increased or reduced diurnal
variations in peak flow is characteristic.
50-B Emphysema shows evidence of airways obstruction but
usually no reversibility. Transfer factor is low due to loss of alveolar
wall space. These 2 features help differential it from asthma.
Hyperinflation (increased TLC) common. Occurs almost exclusively
in smokers.
51-H
The clue here is lung volumes that are half normal
suggesting a lung has been removed. Uncorrected transfer factor
does not take the loss of volume in to account so may also about
half. When it is corrected for volume (ie measures the diffusion
capacity of remaining lung) it may well approach normal values.
Usually some compensation occurs by the remaining lung so
volumes may be more then half.
52-E Restrictive picture (FEVI/FVC ratio > 70%). Globally reduced
lung volumes & reduced inspiratory & expiratory mouth pressures
suggest the cause is neuromuscular disease. Also seen in Guillain
Barre, Motor neuron disease.
53-A FEVI/FVC ratio > 70% suggests restriction. Confirmed by
reduction on TLC. Low transfer factor is common. Restriction also
seen in obesity.

Jen & Adil

Medicbyte
THEME:
54-C Septic arthritis Joint damage can occur within 24 hours of
ones. Usually Staph Aureus & streptococcus. Gonococci important I
young adults. Aspirate immediately for urgent microscopy &
culture, urgent Gram stain. Do an infection screen X-Rays may be
normal initially. Treat with prolonged iv antibiotics, analgesia &
regular aspiration.
55-B
Rheumatoid Arthritis. History & examination point to
diagnosis. Multi system disease (e.g. sicca symptoms here) X-Rays
in later stages show loss of joint space, juxta-articular
osteoporosis, bony erosions + subluxation. Rheumatoid factor
positive in 75%. Manage with physiotherapy, occupational therapy,
analgesia. Consider intra-articular steroids in affected large joints
replacement .
56-D Osteoarthritis. Commonest joint disease. Heberdons nodes.
All investigation usually normal. Joint X-Ray may show loss of joint
space, subchondral sclerosis & cysts & marginal ostcophytes. Treat
with simple analgesia. Weight loss may help. Consider joint
replacement
57-E Actual gout. Hyperuricaemia & deposition of mono sodium
urate crystals. 2% to alcohol diuretics & renal failure. Also trauma,
surgery, starvation, polycythaemia, leukemia chemotherapy,
infection. Serum uric acid not always raised. Diagnosis is by
microscopy of synovial fluid (negatively birefringent crystals). Treat
with regular NSAID or colchichine. Prevention dietary advice,
review drugs allopurinol. Caution do not give allopurinol until 4-6
weeks after an acute attack.
58-A
Reiters syndrome. Triad of urethritis, conjunctivitis &
asymmetrical oligo sero-negative arthritis. Usually a young man
with history of non specific urethritis / dysentery, keratoderma
blennorhagica, mouth ulcers & circinate balance also occur. Treat
with rest & analgesia. Consider disease modifying dugs of recurrent
arthritis (60%) arrange ophthalmoscopy. Recovery may take
months. 60-90% have HLAB27.

Jen & Adil

Medicbyte
THEME:
59-G Typical picture of Primary Biliary Cirrhosis = Progressive non
suppurative cholangio-hepatitis 90% women aged 40-60.
Associated with other autoimmune disease (thyroid, Sjorgens
syndrome). Cholestatic picture. Anti mitochondrial antibody in
95% Liver biopsy diagnostic. Mostly symptomatic treatment aimed
at
pruritis,
varices
malabsorption,
osteomalacia.
Immunosuppresants & liver transplantation have tried, jaundice
develops late. Prognosis 6-7 year survival.

60-F Pancreatic cancer = 1-2 % all malignancy 60% involve head.


Diagnosis-ultrasound ERCP, CT & needles biopsy. Untreated survival
9 weeks. After palliative bypass. 24 weeks.

61-E Hepatitis B virus associated with 80% primary hepatocellular


carcinoma. The clue is his origin Note non-metastatic sings of
malignancy.

62-J Alcoholic hepatitis. Progression to cirrhosis in 90% with these


histological features if continued drinking Liver damage disrupts
control of clotting & glucose.

63-L Sclerosing cholangitis = Short strictures in the intrahepatic &


extrahepatic Biliary system 70% associated with Ulcerative colitis.
Ascending cholangitis & jaundice common Diagnosis ERCP shows
characteristic beading of bile ducts Liver biopsy shows typical onion
skin fibrosis. Usually progresses to secondary Biliary cirrhosis.

Jen & Adil

Medicbyte
THEME:
64-E Angina can sometimes be confused for peptic ulcer disease
as symptoms can occur when eating or at night. The clue here is
exertional pain with a character radiation. ECG may be normal
between attacks. Arrange an exercise test consider coronary
angiography.

65-B Myocardial infarction: Diabetes are prone to arteriovascular


disease. Relative risk of MI = 3-5, CVA =2 Hyocardial is common.
Do an ECG first

66-A Non steroidal anti inflammatory drugs are a common cause


of peptic ulcer disease. Eating relieves duodenal ulcer (hence
weight increase, anaemia necessitates further investigation. Always
do real examination & test for faecal occult blood.

67-H Spontaneous pneumothorax in tall men who smoke is not


uncommon. Due to rupture of a pleural bled, probably congenital
<20% apical spontaneous pneumothorax can be left to resolve if
patient is otherwise healthy. Allow home & repeat CXR in 1 week or
assess sooner of condition deteriorates. If large or causing
respiratory distress, aspirate or insert chest drain 30% recur.

68-F
Aortic disease is a clinical emergency. Always seek
immediate expert help commonly in association with Hypertension.
Aggressive control of blood pressure needed. The danger is
confusion with an MI (thrombolysis can be fatal).

Jen & Adil

Medicbyte
THEME:
69-E

70-F

71-I
Can often mimic physical disease & be admitted as an
emergency. Clues are generalized paraesthesia, bizarre limb
movements & tetany. Depersonalisation can occur (described here
as cotton wool). A hyperventilation test may confirm the diagnosis
by precipitating symptoms.

72-K

73-H

Jen & Adil

Medicbyte
THEME:
74-A Children with sickle cell disease have difficulty handling
capsulated organisms.

75-B
This is legionnaires disease commonly found in air
conditioning of hotels. Also spread by vapour from showers.
Conventions held at such places can cause to endemic spread.
Symptoms are atypical (haematuria, abdominal pain & diarrhea
can occur). CXR shows consolidation in 90%.

76-F Weils disease (Leptospirosis) is spread by contact with rats


urine. Causes fever, jaundice, injected conjunctive. Other myositis,
purpura, bleeding, meningitis, renal failure.

77-L Psittacosis. Diagnose by serology & history.

78-A In nephrotic syndrome there is a hypogammaglouinaelic


state due to profound protein leak. Therefore all diagnosed should
be placed on prophylactic penicillin to protect against potentially
life threatening. Streptococcus Pneumoniae Peritonitis.

Jen & Adil

Medicbyte
THEME:
79-A Partial seizures. Signs & symptoms localized to one part of a
cerebral hemisphere. May become generalized. Aura, automatism &
absence are characteristic but do not always occur together.

80-I

81-F

82-J

83-D

Jen & Adil

Medicbyte
THEME:
84-B

85-A

86-C

87- B / F

Jen & Adil

Medicbyte
THEME:
88-B
89-C Acute dystonia in young people starting neuroleptic therapy
(although is usually of unknown cause). It may manifest as a
hyperextension of the head on the neck, drawing of the eyes
upwards & inability close the mouth (trismus). Treat with
anticholinergic.

90-D
Myotonic dystrophy. Tonic spasm of muscle occurs.
Autosomal dominate, onset age 20-30. Possible features; weak
limbs myotonia, facial muscle wasting. frontal balding, gonadal
atrophy, cardiomyopathy & diabetes mellitus.

91-D
Ducnheine Musular Dystrophy (this is Gowers sign). Sex
linked recessive disorder therefore affects boys. Defect in gene on
Xp21 causing absence of dystrophin. Serum creatine Kinase is
raised at least
40 fold in affected. Few survive beyond early adulthood.

92-L
Long term antiemetic treatment can cause orofacial
dyskinesia (e.g. persistent grimacing chewing), especially in the
elderly. This may persist after withdrawal of drug. A void long term
use of antiemetic.

Jen & Adil

Medicbyte
THEME:
93-E

94-C

95-D

96-A

97-B
Dont forget that a personality disorder is a constant. It may be
recognizable by adolescence & continues through out the
individuals life.

Jen & Adil

Medicbyte
THEME:
98-k this may be seen as a part of the bereavement process &
therefore counseling should be undertaken to ensure that the
bereavement is allowed to go through the full process.

99-J This is an obsessional-compulsive disorder. Remember a


normal activity is disrupting everyday life style.

100-L This man has Delirium Tremens.

101-E
He is demonstrating some of Scneiders First Rank
Symptoms.

102-I Various techniques may be used Sometimes the individual


is just asked to face the anxiety provoking situation as is often the
case in fear of flying.

Jen & Adil

Medicbyte
THEME:
103-A

104-D Hyperuricaemia & hyperkalaemia can occur where cell


turnover is high.

105-C Characteristic features of neurological TB in CSF.

106-K Adrenocortical insufficiency causes Na+ loss, potassium &


mild acidosis. Hypercalcaemia & Eosinophilia can also occur. In the
UK,80% is idiopathic (probably autoimmune) & associated with
other autoimmune disease (Graves diabetes mellitus, pernicious
anaemia).

107-G

Jen & Adil

Medicbyte
THEME:
108-B

109-C

110-G

111-A

112-H

Jen & Adil

Medicbyte
THEME:
113-F This child demonstrates Gowers sign. This occurs if there is
a proximal muscle weakness, in this case Duchenne Muscular
Dystrophy.

114-A
Remember the non organic causes for psychiatric
symptoms. Thyroid function tests are routinely available. Of the
options given here, this picture may also be caused by renal failure
or cardiac failure where weight gain is due to fluid retention. Thus
strictly speaking, options E & B/C would also apply. Remember
clinical situations rarely have a clearest answer. It is important to
think why you are ordering particular investigations.

115-D This is Epstein Barr Virus Infection (kissing disease)

116-B Bacterial endocarditis usually occurs in the right heart in


intravenous drug abusers. listen for a murmur. Echocardiography
should pick up most lesions. > 3mm. Rash = Splinter
hemorrhages. ESR/CRP will be raised & can be used to monitor
treatment. Look for Roths spots & Janeway lesions. Treat with high
dose prolonged antibiotics.

117-K Diabetes Mellitus can present with fatigue as can just about
any illness. The clue here is polyuria. Urine dipstick for glucose
should be followed by appropriate testing of venous blood & oral
glucose test if necessary.

Jen & Adil

Medicbyte
THEME:
118-B
B6 = pyridoxine Deficiency occurs secondary to
malabsorption alcoholism & Isoniazid therapy. Mental confusion,
glossits peripheral neuropathy & dry skin lesions occur.

119-H Folic acid is given to pregnant mothers as helps in the


prevention of neural tube defects. Deficiency occurs in diets lacking
in fresh vegetables & meat, malabsorption & with methotrexate
/trimethoprim. In pregnancy, demand for Folate increases.

120-C Scurvy

121-E

122-I This is very rare!!

123-L Most thiamine deficiency in the UK occurs in malnourished


alcoholics.
Wernickes
encephalopathy
(nystagmus,
ataxia
confusion) proceeds to Korsakovs syndrome (described here) if
untreated. In developing countries, thiamine deficiency caused
BeriBeri. Diagnose by high erythrocyte transketolase activity &
blood pyruvate.

Jen & Adil

Medicbyte
THEME:
124-C

125-A

126-D This is also known as primary amenorrhoea. There is


hypergonadotrophic hypogonadism where underdevelopment of
gonads causes increased gonadotrophin secretion by negative
feedback.

127-F
This is also known as secondary amenorrhoea.
Hypothalamic pituitary under secretion occurs secondary to e.g.
stress, weight loss, severe systemic illness.

128-F This is describing a young lady with anorexia nervosa &


secondary amenorrhoea.

Jen & Adil

Medicbyte
THEME:
129-B
Gonococcus can infect any columnar epithelial surface
(rectum, cervix, pharynx, conjunctiva) but not vagina which is
squamous, 50% have concurrent, Chlamydia trachomatis. Females
are often asymptomatic. Complications, Salpingitis, pustular rash
on hands & feet, endocarditis.

130-D

131-E IUCD problem expulsion, ectopic pregnancy, PID, infertility,


menorrhagia, abdominal cramps uterine perforation. A Israeli is a
recognized colonizing organism.

132-C
Chlamydia causes up to 70% of PID Most is sexually
acquired. Subacute infection with vague or no symptoms is easily
missed. Laporoscopy is often required to confirm the diagnosis.
Acute disease can occur with severe systemic symptoms (fever,
abdominal pain & shock).

133-A Toxic shock syndrome occurs due to Staphylococcus Aureus


toxin release from tampons left in situ. Features: fever, rash,
desquamating rash, diarrhea & shock.

Jen & Adil

Medicbyte
THEME:
134-A

135-F

136-C

137-J

138-I

Jen & Adil

Medicbyte
THEME:
139-A
Heavy protein leak n nephritic syndrome (loss of
immunoglobuin in urine) increases risk of pneumococcal infection.

140-A Recurrent sickle cell crises cause splenic infarction & may
result in autospleenectomy. The spleen deals with encapsulated
organisms such as pneumococcus.

141-D This is pneumocystis carinii prophylaxis.

142-E The treatment dosage with Trimethroprim is different from


the prophylaxis dosage.

143-F High mortality from bacterial meningitis justifies contact


chemoprophylaxis with Rifampicin.

144-K
Amoxycillin & Gentamycin are recommended as
prophylaxis for genitourinary procedures in patients with prosthetic
heart valves abnormal valves or congenital defects such as PDA.
Dental procedures also carry a risk & oral Amoxycillin is
recommended. Always consider antibiotic prophylaxis in these
patients if undergoing invasive procedures.

Jen & Adil

Medicbyte
THEME:
145-B

146-D

147-F This is due to Haemolytic Uraemic syndrome which is


associated with E. coli infection

148-G

149-E This is due to an E. coli UTI

Jen & Adil

Medicbyte
THEME:
150-G This is Lyme disease.

151-H This is Reiters syndrome occurring secondary to gut


infection (Shigella, Salmonella, Yersinia, Campvolbacter), Males 20
x more affected than females. Trial = seronegative reactive
arthritis, non specific urethritis & conjunctivitis. If acute phase,
stool culture may be helpful. If the acute illness has passed,
diagnosis is based on history, raised ESR, negative antibody screen
& sterile joint aspirates (i.e. by exclusion) 60% are HLA positive.
60% develop recurrent arthritis.

152-J This is rheumatic fever due to post strep infection

153-K This is gout.

154-C Ankylosing spondylits. Occurs in late teenage / early adult


years. Sacroileitis (+ radiating pain) polyarthritis, plantar fascitis
(heel pain) & iritis can occur. X-Ray changes occur as disease
progresses fusion of sacroiliac joints, tramline appearance,
syndesmophyte formation & squaring of vertebrae. Treat with
analgesia & physiotherapy. The key is to keep the patient
exercising.

Jen & Adil

Medicbyte
THEME:
155-D Mono-spot test. This girl has Haemolytic anaemia
secondarily to EBV infection. Therefore a mono-spot or a paulbunnel test should be ordered.

156-E A Coombs test will be strongly positive if the hemolytic


anaemia is due to Rhesus disease & only weakly positive if due to
ABO incompatibility.

157-G The cause is hereditary spherocytosis & it is an autosomal


dominant condition. Blood film will show spherocytes.

158-C It is Paroxysmal Nocturnal Hemoglobinuria. Hams test is


used in its diagnosis.

159-I A thick & thin blood film is mandatory for anyone that has
traveled to a malaria endemic area & returns with a pyrexia.

Jen & Adil

Medicbyte
THEME:
This is an example of the sort of practical situations faced in real clinical
practice. First line management will always include stabilization of the
patient medically, but it is just as important to ensure the patient receives
the appropriate care thereafter. The new examination format is trying to
bring such scenarios to the PLAB paper. The advantage of this is that
candidates are more focused on learning practical points.

160-B

161-E

162-G Organic brain syndrome. Characteristic sign is clouding of


consciousness. There may be features of other psychiatric disease
but these resolve on withdrawal of opiates in this case. Causes:
Drugs, infection, hypoxia alcohol withdrawal. This patient can be
reassured.

163-E The important fact to point out here is that treatment of


anorexia nervosa should not involve enforced measures unless
absolutely necessary. Listen to the patient and refer to a
psychiatrist. Management involves counseling of family & patient,
cognitive therapy & dietetic help in setting weight targets.

164-H The mental health act allows detention of a patient for


specified amounts of time by various personnel for patients
considered at serious risk to themselves or others. Detention of a
patient already in hospital for 72 hours can be authorized by the
doctor in charge (or deputy) as long as there is liaison with hospital
administration & a clear plan of where the patient goes afterwards
i.e. psychiatric referral.

Jen & Adil

Medicbyte
THEME
165-E This is placenta praevia. The main symptom of placenta
praevia is painless vaginal bleeding. Ultrasound is used to diagnose
placental position.

166-F This is abruprio placenta. In contrast to placental praevia,


placental abruption presents with vaginal bleeding which is painful
& there is increased uterine tone

167-G Not enough information is provided by clinical examination


alone & the first step would be CTG monitoring.

Jen & Adil

Medicbyte
THEME :
168-B
Coeliac disease. This would be the first test undertaken
followed by small bowel biopsy

169-G This is crohns diseases.

170-H This is secondary lactose intolerance. Some children need


a trial of elemental milk that does not need the lactase enzyme to
break it down.

171-J This is acute pancreatitis.

172-L This lady has acute bowel obstruction.

Jen & Adil

Medicbyte
THEME:
These are common ECG patterns, if available, always compare with
an old ECG.
173-L
ECG signs do not always occur in PE but the S 1, Q 3, T
3 pattern plus clinical history points to the diagnosis. Atrial
fibrillation can occur with many conditions & is non-specific. If new
onset & no obvious cardiac cause, be alert for e.g. pneumonia,
pulmonary embolism, metabolic upset.

174-A
this is the threshold for diagnosing ischaemic heart
disease on treadmill testing.

175-C Digoxin can also cause almost any arrhythmia (SVT with AV
block is suggestive). The reverse tick sign indicative of current
therapy (not necessarily toxicity) Digoxin toxicity is precipitated by
hypokalaemia & anything interfering with renal excretion.

176-k In MI, within hours, T waves become tall & ST segments


begin to rise. Within 24 hours, STs fall & Ts begin to invert, Within
a few days, pathological Q waves appear & persist. T wave changes
may resolve.

177-J Typical ECG appearance of hyperkalaemia.

Jen & Adil

Medicbyte
THEME:
178-F clue to hypothermia is J waves. Complexes may be small.
Hypothermia initially causes tachycardia followed by J waves,
increased PR interval hypotension & occasionally V-fib.

179-J
The clue to acute pericarditis is the development of
characteristic upward saddle shaped concavity of the ST
segments.

180-G Small complexes indicate pericardial effusion.

181-I This is one of the commonest way to look for LVH on an


ECG, often secondary to hypertension.

182-K Pneumonia is a recognized cause of Atrial fibrillation,


usually self resolving.

Jen & Adil

Medicbyte
THEME
183-A
Stress headache. Commonest of headache
Important to listen. The band like ache is a clue

in

UK.

184-K Meningococcal Meningitis Rapidly fatal. Treat immediately


with high does iv benzylpenicillin. Then if no signs of raised
intracranial
pressure
(vomiting,
seizure,
mood,
change,
bradycardia, impaired consciousness & respiration, papilloedema),
& no local CNS signs, LP should be done without delay. If either of
these features exist, arrange urgent CT head to exclude apace
occupying lesion before LP look for source of infection Remember
contract prophylaxis.
185-E Migraine. Common migraine is not usually associated with
aura. Family history is usual.

186-B Giant Cell arteritis. Arteries are usually tender. Take an


immediate ESR & start high does steroids with out delay or there is
a risk of blindness. A temporal artery biopsy will confirm the
diagnosis (arrange for a few days after treatment has started)
187-D Sub-arachnoid Hemorrhage 80% due to ruptured berry
aneurysm 30% rebleed within days. Do urgent CT to rule out SOL
then LP CSF shows xanthoeromia (yellow) & blood. If diagnosis
confirmed & patient conscious consult surgical team for possible
angiography & clipping. Consider evacuation if life threatening. ECG
changes can occur (T inversion, V waves, arrhythmia).

188-C
Being intracranial hypertension. Affects young obese
woman. Presents with raised ICP usually no cause found. But
important to rule out SQL so do CT head first & full examination for
focal signs. Look for papilloedema. Treatment is with repeated LP,
dexamethasone, consider shunt.
189-I Sinusitis. Dont forget the common causes.

Jen & Adil

Medicbyte
THEME:
190-B Wells disease Leptospira Icterohaemorrhagiae. Spread by
contract with infected rats urine (swimming, lab & sewer workers).
Causes jaundice, myositis, meningitis, bleeding & renal failure.
Diagnosis by complement fixation tests & culture of blood, urine &
CSF.

191-E Neisseria meningitides. See previous question. Presentation


often indistinguishable from other bacterial infection. Causes 70%
of all meningitis out with neonatal.

192-L Cryptococcal (fungal) meningitis in established AIDS can


have a rapid course. Diagnosis India Stain & Cryptococcal antigen
in CSF. Aids increase risk of meningitis from all usual organism plus
atypical e.g. fungi, TB, Listeria. Also, pre conversion acute aseptic
meningitis can occur (meningism, cranial never palsies (esp. 5,7,8)
& long tract signs).

193-C Clinical meningitis occurs in 5% of mumps. 30% do not


show parotid swelling. Diagnosis clinical. If in doubt, serology for 4
x rise in antibody titers between acute & convalescent phase is
diagnostic. Treat symptomatically (analgesia, nutrition). Prevent by
immunization.

194-H
TB meningitis usually presents with a vague gradual
history (unless immuno-compromised when atypical & fulminant
symptoms can occur). CSF is turbid with a high mononuclear cell
count, high protein & low glucose.

Jen & Adil

Medicbyte
THEME:
195-A Beta blockers are used standard therapy for angina. They
are contraindicated in asthma

196-K maximal therapy for angina is likely to include a calcium


antagonist. Verapamil causes constipation. Also contraindicated in
heart failure.

197-I Early use of an ACE inhibitor post MI (SAVE, ISIS 4 &


GISSIS 3 trial) reduces mortality & morbidity & morbidity. Baseline
renal function should be checked & monitored electrolytes.

198-J Her chronic disease is likely to be epilepsy given that she


has gum hyperplasia which occurs commonly with Phenytoin
(also seen in scurvy, nifedipine & acute myeloid leukemia).
Phenytoin therapy must be monitored.

199-L He will be on Warfarin to prevent clot formation. Warfarin


effect is increased by alcohol. Thus advise against alcohol when on
treatment as there is a risk of bleeding.

200-E The diagnosis is infectious mononucleosis. Never give


Ampicillin if this is suspected.

Jen & Adil

Medicbyte
THEME:
Note that these are all common drug interactions. They may at first seem
difficult but the answer are provided simply by a recognition of which drug
are used in treatment of which disorders. The Question illustrates the
potentially hazardous effects of pharmacy in everyday practice. Always
consider possible interactions when prescribing a new drug, especially in
the elderly & those with renal or liver disease.

201-H He is on Digoxin for Atrial fibrillation. His GP has added


Frusemide. Hypokalaemia secondary to Frusemide precipitates
Digoxin toxicity. Xanthopsia (a feature of Digoxin toxicity.
Xanthopsia ( a feature of Digoxin toxicity)=seeing yellow.

202-G She is on lithium for mania. A diuretic (bendrofluazide


usually) has been added for hypertension. Thiazide diuretics inhibit
renal conversation of sodium & lithium is retained instead.
Therapeutic range is narrow & levels must be monitored. Signs
here are of lithium toxicity.

203-A She is likely to e on oral contraception. The clue is that she


is career woman. Antibiotics, barbiturates, Carbamizipine,
Phenytoin & Rifampicin reduce OCP efficacy. Also any
gastrointestinal
illness
causing
diarrhea,
vomiting
or
malabsorption.

204-B A Persistent arrhythmia which requires DC cardioversion


is usually Atrial fibrillation. Before cardioversion, is standard to
anticoagulate for 4 weeks with Warfarin. Anti-tuberculous therapy
includes Rifampicin which is an enzyme inducers (increase Warfarin
metabolism & thus reduces effect). Failure of adequate
anticoagulation has resulted in intracardiac clot & subsequent
stroke.

205-L These drugs are nephrotoxic.

Jen & Adil

Medicbyte
THEME:
206-A or B PFTs Note TFTs would also be can appropriate &
practice you monitor both before & during treatment. Amiodarone
can causes any form of thyroid disturbance, corneal deposit,
alveolitis (CXR may not pick up early disease), liver disease &
pigmentation.

207-C Can cause renal failure & is not usually used if baseline
renal function abnormal.

208-E Narrow therapeutic range. Toxicity arrhythmia, anxiety,


tremor & seizures.

209-C
Watch for hypokalaemia, hypocalcaemia. Can cause
nephrotoxicity & precipitate other drug side effects.

210-E
Gentamycin
monitored. Peak (1/2
Trough (just before
deafness, nystagmus,

Jen & Adil

is highly nephrotoxic & levels must be


hour post iv does) should be 9-18 um01/1.
does) 2umol01/1. Toxicity tinnitus,
vertigo, renal.

Medicbyte
THEME:
211-D Temporal lobe epilepsy (Complex partial seizure) aura
followed by automatism & absence. Focal changes are sometimes
but not always seen on EEG
212-B Postural Hypotension due to over treatment with diuretics.
Electrolyte screen should also be checked.

213-C Aortic Stenosis ECG might show signs of LVH & strain.

214-A Hysterical hyperventilation ABGs would show low CO2 &


raise 02 during an attack. Attacks may mimic grand mal seizures
(jerking, loss of continence) but limb movements are usually
bizarre & tetany with non specific paraesthesia suggest diagnosis.
A 2 minute hyperventilation test may help. Although this patient
says she remembers nothing, consciousness is usually retained.
215-A Signs of raised intracranial pressure. Also vomiting, seizure,
mood change, bradycardia, impaired consciousness & respiration,
papilloedema. Space occupying lesion needs to be ruled out by CT
head. If LP is performed in the presence of raised ICP (intracranial
pressure), there is a risk of coning (herniation of uncus downwards
through tentorial hiatus). This can be rapidly fatal.
216-G Carotid Sinus syndrome. Due to excessive carotid sinus
sensitivity. Symptoms can be reproduced by massaging the carotid
(with ECG monitoring & resuscitation facilities nearby). Carotid
Sinus Massage can be tired in the treatment of supraventricular
tachycardia.

217-J Hypoglycemia can present with shaking, abdominal pain,


sweating & hunger followed by loss of consciousness (uncommon)
& seizure. Usually occurs in treated diabetics. Do not label as
epilepsy.

Jen & Adil

Medicbyte
THEME:
218-B Alzheimers dementia = 62% of dementia. 70% are women.
Senile plaques & neurofibrillary tangles in cortex. Progress is
relentless. Mean survival 5-7 years. Good support essential. This
patient has no family so history is unlikely from someone who
knows her.

219-F Wernicks encephalopathy. Due to thiamine deficiency,


usually in alcoholics. Opthalmoplegia also common. Treat urgently
with thiamine to prevent development of Korsakoffs syndrome
(confabulation & inability to form new memories). Tests red cell
transketolase reduced or plasma pyruvate increased.

220-H Uraemic. Polypharmacy must be monitored. All these drugs


in everyday use are potentially nephrotoxic, especially in the
elderly. When prescribing a drug, always consider potential
interactions.

221-A Most likely caused is sepsis (indwelling catheter,


pneumonia, infected bedsores). Screen consists of swabs,
urinalysis, blood cultures, ,CRP,+ stool culture/CXR/ sputum (if
available). Always consider effects of polypharmacy. Dehydration,
constipation, arrhythmia, hypoxia & cardiac failure can also cause
confusion in the elderly (although fever unlikely in these cases).

222-I Post ictal confusion is common. The diagnosis of epilepsy is


usually based on a good history & is only made when organic
causes have been excluded (e.g. alcohol withdrawal, hypoxia) as
there are important implications for driving, insurance &
employment. ECG & CT are usually performed but ECG may be
normal & CT is mainly to rule out space occupying lesions.

Jen & Adil

Medicbyte
THEME:
Blood films is useful in the investigation of all hematological
disease.
223-L pernicious Anaemia. Autoimmunity to parietal cells causes
lack of gastric intrinsic factor & malabsorption of B12. Associated
with atrophic gastritis & other autoimmune disease. Ca stomach is
another association so book endoscopy. Investigation: Blood film
(macrocytic), Serum B12 (low). During cell antibodies (in 80%).
Optional: Schilling test (occas done) & bone marrow
(megaloblastic). During Rx with hydroxycobalamin give iron
supplements & watch for hypokalaemia. Exchange transfusion may
be needed.

224-I Iron deficiency anaemia. Blood film will show hypochromic,


microcytic picture & target cells. Total serum iron & ferritin
reduced. Total Iron binding capacity increased.

225-A Disseminated Intravascular coagulation seen in obstetric


complication/ sepsis/ trauma & malignancy. TT, PT, & APTT (all
increased). Rapid reduction in platelets & Hb. Film shows
fragmented RBS & schistocytes. Measure FDPs (increased). Treat
with FFP & seek expert advice.

226-E Multiple myeloma. Plasma cell neoplasm. Diagnose by


urine/ serum electrophoresis (for monoclonal band), Bence Jones
protein in urine & bony XRs (pepperpot skull). Check renal
function, calcium, fundoscopy (hyperviscosity). Do not diagnose on
basis of paraprotein, alone which can occur in other conditions.
Treatment = supportive & chemotherapy.

227-C Beta thalassaemia minor. Heterozygote state. Diagnosis.


MCV<750, mild anaemia (Hb 9-11 g/dl)and HbA2> 3.5%. Usually
well tolerated Raised ferritin & reduced TBIC differentiate this from
iron deficiency anaemia.

Jen & Adil

Medicbyte

THEME:
228-C Diabetes Mellitus. Usually juvenile onset. Associated with
other autoimmune disease, HLA DR3 & DR4, patient always need
some insulin, even if not eating or other illness. Diagnosis : 2 x
fasting blood glucose + oral glucose tolerance test. A dipstick of
urine will alter the physician but will not diagnose diabetes.
229-F
Graves disease. Associated with other autoimmune
disease (e.g. insulin dependant diabetes mellitus, pernicious
anaemia). Eye signs (Proptosis/exophthalmos/opthalmoplegia),
pretibial myxodema & thyroid acropachy only occur in Graves.
Always test visual acuity & seek immediate advice if reduced.
Treatment options-carbimazole (watch for agranulocytosis),
thyroidectomy, radio-iodine

230-A Cushings disease is due to bilateral adrenal hyperplasia


(compare Cushings syndrome = due to glucocorticoid excess).
Affects 30-50. Diagnosis: a) Raised 24 hours urinary cortisol x 3
(normal < 700) nmol / 24 hr confirms cortisol excess. B) High does
dexamethasone test cause some but normal suppression of plasma
cortisol, c) ACTH is raised (but<250 ng/l/d) Metyrapone test
produce excessive urinary steroid-b, c, d confirm Cushings
disease. Screen for diabetes.
231-B
Addisons disease. Autoimmune in 80% Primary
hypoadrenal cortex (reduced gluco/mineralocorticoid & sex
steroids)
leading
to
hyperkalaemia,
hyponatraemia,
hypercalcaemia, hypoglycemia, hyperuricaemia. Eosinophilia &
metabolic acidosis also occur. Reflex rise in ACTH causes
pigmentation of palmar creases & buccal mucosa. Diagnosis short
ACTH stimulation produce absent or reduced cortisol response
(confirmed by long ACTH suppression test which will exclude
adrenal suppression due to steroids). Plasma ACTH>80ng/l
associated with low cortisol is also diagnostic
232-H Acromegaly. Rare. Due to hypersecretion of GH from a
pituitary
adenoma.
Complications
heart
failure
diabetes,
hypertension, cardiomyopathy. Comparison with old photographs
provides diagnosis. Other test visual acuity, ECG, basal serum
prolactin, growth hormone level, CT pituitary fossa. Look for
evidence of other hormone involvement (T4, testosterone, OGTT,
Triple stimulation test if hypopituitarism suspected of surgery
likely). Rx is surgical + radiation/bromocriptene.

Jen & Adil

Medicbyte
THEME:
Seek expert help in the management of all cases as therapy itself
can have risks. Most antitoxins have to be ordered specially
Naloxone & Flumazenil are usually safe to give.
233-B Atropine is useful for sever bradycardia or hypotension
234-C Rapid iv bolus useful if respiratory
235-I
If not normally on anticoagulation & not bleeding,
phytomenadione ( vitamin K) is used
236-E These inactivate Digoxin but are not readily available.
237-H Opiates are present in many analgesics. The cardinal sign
in the absence of focal neurological deficit is bilateral pinpoint
pupils. Methadone is an Options for high dose users.
238-G intravenous infusion chelates iron
239-K
Paracetamol poisoning. Management options: gastric
emptying, N acetyleysteine/ dextrose regime, monitor blood
glucose, clotting & liver enzymes (risk of liver failure) &
electrolytes. Low threshold for ITU. Treatment is guided by blood
paracetamol levels, time since ingestion & blood parameters. A
Paracetamol treatment chart is routinely used on most wards.
Transfer to ITU if increased intracranial pressure, encephalopathy,
unresponsive INR, renal impairment, blood pH<7.3, systolic
BP<80.
240-F This was caused by Botulinum toxin found in tinned foods.
Causes diplopia, blurred vision, photophobia, ataxia, pseudobulbar
palsy & rapid Cardiorespiratory failure. Treat in ITU with expert
help. Intubate immediately.
241-D IV infusion of dicobalt edatate followed by dextrose.
242-J Fresh frozen plasma & Vitamin K. if it is essential that the
patient says anticoagulated, use FFP & seek expert help. Treat
major bleeding with high dose iv Vitamin K.

Jen & Adil

Medicbyte
THEME:
243-B Coning occurs due to rising ICP. The uncus (of the temporal lobe)
is pushed through the tentorial hiatus & is squeezed alongside the
brainstem. Both brainstem function & 3 rd nerve are affected, initially
unilaterally, then bilaterally. Posture is initially decorticate, then
decerebrate. 6th nerve palsy may occur but falsely localize the problem as
it has a long intracranial course. Coning can also occur after LP in ICP.
Manage by burr hole surgery (before papillary dilation), O 2 & mannitol to
reduce edema.

244-A Tension pneumothorax occurs because air drawn into the pleural
space (e.g. flair rib) on inspiration and cannot escape. The mediastinum is
pushed to the opposite side. Treatment is with immediate aspiration with
large bore needle (before any other investigation). Cardiorespiratory arrest
will occur if untreated.

245-C DIC is an activation of coagulation (malignancy, trauma, infection,


obstetric) causing clotting factor & platelet consumption. Fibrin strands
accumulate throughout small vessels & haemolyes red blood cells.
Extensive bruising, bleeding, renal failure, gangrene may occur. Blood film
shows fragmented red blood cells & schistocytes. Measure FDPs
(increased) Treat causes & seek expert advice.

246-L Anaphylactic shock (ABO incompatibility): It is likely that he was


transfused (although he may have bled from a peptic ulcer, do a rectal).
Treat mild reactions (itching, bronchospasm, slow onset fever) with
hydrocortisone, chlorphenyramine & consider stopping or slowing
transfusion. Always check identity. Heart failure can occur if transfusion
too rapid. Oral Frusemide is often given prophylactically with each unit in
those at risk. Rapid spike of temperature at start indicates anaphylaxis, so
stop transfusion. Treat major anaphylaxis by stopping transfusion still
required, seek expert advice.

247-E Massive E often presents few days post op with any of dyspnoea,
pleuritic pain, shock, cyanosis, haemoptysis, gallop rhythm, raised JVP
loud P2, pleural rub. Look for source (DVT). ECG may show S-1, Q-3, T-3
pattern (described elsewhere), ABGs low PaO2. If life threatening,
consider surgery. Otherwise treat with oxygen, opiates, iv heparin (then
Warfarin). Investigate according to local protocol (V/Q, pulmonary
angiography or spiral CT).

Jen & Adil

Medicbyte
THEME:
248-B Prostatic carcinoma is most likely. Onset usually insidious.
Bone pain/tenderness(as in this patient) indicates metastasis (XR
will show sclerotic or lytic lesions) . Diagnosis Rectal examination is
essential-firm irregular prostate with no median sulcus. Also raised
Prostatic specific antigen, bone scan & Prostatic ultrasound.
249-A
Nephrotic syndrome (proteinuria > 3.6 g/24hr) causes
hypoalbuminaemia & edema, often with raised cholesterol.
Causes : all glomerulonephritides, minimal change disease,
vasculitis, SLE, diabetes mellitus, amyloid, drugs & allergies.
Investigation: 24 hour urine for creatinine clearance & protein.
Microscopy for casts & red blood cells. Renal Biopsy avoid in
children unless necessary. Check renal function, albumin,
cholesterol,
complement,
autoantibodies.
Complications

thrombosis (pulmonary embolism, renal vein, deep venous) ,


pneumococcal infection, raised cholesterol, renal failure.
250-E Rule out urinary infection.

251-D Polycystic kidney disease. Autosomal dominant in adults


(choromosome 16). Common cause of chronic renal failure. May
present as frank haematuria, urine infection, abdominal mass or
hypertension. Associated with cerebral artery aneurysm &
subarachnoid hemorrhage, pancreas & liver cysts. Diagnosis: renal
ultrasound.

252-G 90% renal stones are radioopaque thus do KUB in all


(pregnancy). Consider intravenous urography & 24-hour urine
collections for oxalate, calcium phosphate, urate (components of
stones). Treat with analgesia , sieve urine & increase fluids. Most
pass spontaneously. Seek urological help if infection or obstruction
occurs.

Jen & Adil

Medicbyte
THEME:
253-D

254-F The lacy patterning is called wickhams striae

255-H

256-J

257-K

Jen & Adil

Medicbyte
THEME: missing theme
271-G
272-I
273-B
274-D
275-E

276-k

277-I

278-e

Jen & Adil

THEME

Medicbyte
THEME:
280-H

281-K

282-J

283-F

Jen & Adil

Medicbyte
THEME:
284-E It is important not to miss diagnosis in this patient as delay
in treatment may lead to a permanent visual loss. Female
preponderance in giant cell arteritis is characteristic as well as the
age of onset (50-90 years). Other features such as scalp
tenderness & thickened temporal arteries could also be found.

285-C Local release of 5-HT in the area close to the superficial


temporal artery causes symptoms in this middle-aged man. Cluster
typically last up to 12 weeks with a period free of symptoms for
approximately 1 year. In addition, there may be Horner syndrome,
which remains permanent in 5 % of patients.

286-A This woman probably has a difficult life trying to balance


between her job & the family. Tension headache is often associated
with continuous street. Pain may also be described as dull or like a
pressure & is typically experienced every day for a long period of
time (years).

287-B Classical migraine is described be a trial of paroxysmal


headache, vomiting & focal neurological events. That patient is said
to have common migraine, as she does not complain of vomiting. It
is important not to mistake her neurological symptoms with
Jacksonian epilepsy, which is not usually followed by throbbing
headache.

Jen & Adil

Medicbyte
THEME:
288-C The child has typical follicular tonsillitis. The pus exuding
from the crypts is characteristic. Earache in that case is usually
referred.

289-B The clues to the diagnosis in this young man are malaise,
which is often more severe than in acute tonsillitis, yellow
membranes on the tonsils & enlarged lymph nodes. Diagnosis is
confirmed by Lymphocytosis in the peripheral blood & positive PaulBunnel test.

290-A Diphtheria has become rare in UK but it is emerging again


in some parts of Eastern Europe & South East Asia. Hence, recent
travel to these continents is characteristic. Diphtheria presents
sometimes as an emergency when sever edema & membranes lead
to airway obstruction & asphyxia. Firmly attached grey membranes
that are difficult to remove & that leave a bleeding surface after &
attempt to peel theme off point towards the diagnosis.
291-D It is difficult to make a mistake dealing with such a bright
clinical picture. In addition these patients have a characteristic
tongue, which is described as strawberry & cream.

292-F Agranulocytosis is a recognized said effects of carbimazole,


which is used for the treatment of thyrotoxicosis. Patients receiving
this drug should be warned, therefore, to see their GP if, sore
throat develops.

Jen & Adil

Medicbyte
THEME:
293-B As can be assumed from the symptoms & past medical
history this young man is likely to have type 1 diabetes mellitus
(ketoacidosis occurs almost always in absolute insulin deficiency).
Therefore, insulin is the only option.

294-C Tolbutamide is the drug of choice in that case. It is a firstgeneration short acting sulphonylureas drug with mild effect.
Beside it is very well tolerated & has a low risk of inducing
hypglycaemia.

295-D Metformin (biguanide) has a different mechanism of action


from sulphonylureas. It increase insulin sensitivity & impairs
glucose absorption in the gut. Hence, it does not cause inducing
hypoglycemia & professional drivers can use it. In addition, its
administration is not usually followed by a rice in body weight and,
therefore, it may be preferred for the obese individuals.

296-A Chlorpropamid may be tried I that case. It is a long acting


first-generation sulphonylurea drug with a biological half-life of
about 36 hours. However the doctor prescribing this drug should be
aware that it can cause severe & prolonged hypoglycemia.

Jen & Adil

Medicbyte
THEME:
297-G

298-I

299-B

300-D

301-E

Jen & Adil

Medicbyte
THEME:
302-O

303-G

304-A

305-I

306-N

Jen & Adil

Medicbyte
PLAB COURSES.
I was asked to write has section by a number of doctors
whom have had both good & bad experiences of UK PLAB
courses. There are good & bad courses available in the
United Kingdom.
For PLAB part one Course I Just have one piece of advice:I believe that it is always best to be taught medicine by
medical doctor. After all, the clinical subjects at medical
school are taught by medias. Therefore, I do not think
postgraduate teaching should be any different
Therefore if you wish to be taught well, then remember to
ask the qualification of the main course teacher. There are
some courses in the United Kingdom that do not follow this
fundamental principle-dont be influenced by cost!!
ONLINE PLAB TUTORIALS.
Join our free online tutorial session. We provide free online
Q-and practical points for the PLAB examination-you will not
find these in textbooks!! Medicbyte is the first organisation to
have utilised this method of online teaching
If you register with Medicbyte.co.uk you will get these Pearls
for the PLAB examination sent to your email address.
THE MEDICBYTE PART ON COURSE.
Why not for one of our PLAB courses ? Further information
about the course (dates/venues/prices) are available on the
website.
For
queries
about
the
course
email:
course@medicbyte.co.uk

Visit our website to find out more details about PLAB


resources for the examination.
Geriatric medicine
Confusion
Ethics of care of the elderly
Elderly pharmacotherapy
Oncology
This is a topic that lends itself to a number of openings in the
EMQs

Jen & Adil

Medicbyte
There is also an overlap of topics in to the surgery section of
the examination .
Risk factor for cancer
Treatment principles (not detailed)
Cardiovascular system.
Breathlessness
Chest pain.
Heart murmurs linked to history & examination
The cardiac systematic examination linked
pathology
Pulse, Blood pressure
The investigation of cardiac pt
Acute care of the cardiac patient
Pericarditis

to

disease

Nephrology
Acute & chronic renal failure
Haematuria & proteinuria
Hypertension
Infections, trauma, oncology
Gastrointestinal system.
Haematemesis
Diarrhoea
Weight loss
CA (oesophageal, gastric, small & large bowel & anal )
Crohns & Ulcerative Colitis
Coeliac disease
Impact of poor nutrition on disease.
Haematology
See Oncology
Clotting defects & their management
Anaemias (causes/prevention/management)
Endocrinology
Pituitay problems
Problems with the Hypothalamotract leading of disease.

Jen & Adil

Medicbyte
Dermatology
Causes of the common rashes, their link to infections
Candidates should also concentrate on conditions that have a
multi-system manifestation & can effect the skin.
This chapter was written to help guide students on what
knowledge is required for the PLAB part one examination.
This list is not exhaustive & has not been verified by the
GMC. Therefore we cannot endorse it as the definitive list for
the examination. However it is meant as a guide &
candidates will do well to stick to the main areas.
When undertaking clinical attachments or on working in the
medical practice you should consider how you would manage
the common conditions that present to hospital. You must
understand the important points in disease management. We
have summarised these important points in to the Medicbyte
EMQ Revision Principle this is given below.
THE MEDICBYTE EMQ REVISION PRINCIPLE
How did the diagnostic process come about?
What investigation were undertaken & why?
What management decisions were made & why?
What is the prognosis for his condition?
Could this conditions have been prevented & are there any
preventative public health schemes in this place?
When you have assimilated the above Q-then you can go to
the reference text & learn about the conditions.
Incorporating this methodology in to your learning will
ensure that you use your time efficiently.
Candidates should remember that there medical care is best
given in a multidisciplinary manner & as such the
examination will be attuned to this concept. Therefore in
your everyday work consider the Q-what other service will
be of value in dealing with this patients conditions. This
change is reflected by the type & nature of EMQs for the
PLAB part 1. This concept is also portrayed in the Q-in this
book.
General Medicine
This is the most important section of the examination & so
the candidates will do well to concentrate on this area. You
should know about the presentation of the common
conditions in the United Kingdom & how these should be
investigates.
Infectious diseases
The link of microbiological organisms to the certain risk
factor in humans.

Jen & Adil

Medicbyte
Candidates should know of the link between a predisposition
to illness & infection;eg. HIV infection
Appropriate drug therapy & sensitivity of organisms
Immunisation & peryention of infections in populations.
Neurology
Weakness
Myalgia.
Headaches
Ataxia
Epilepsy-history, diagnosis, causes, prognosis, investigation.
Drug side effects.
Respiratory.
Breathlessness
Cyanosis
Management of asthma (remember the possibility of a
pneumothorax)
Acute
respiratory
conditions-pneumothorax;
haemomediastinum.
Surgery
This sections can be divided into acute & chronic care.
The candidates should concentrate on gastrointestinal
conditions & their management.
Acute care-Acute emergencies
Oncology-risk factor, metastatic spread (where? When? How?
)

Jen & Adil

Medicbyte
Obstetrics & Gynaecology
The concept of the high risk delivery
Pre natal, peri-natal & postnatal management in UK
Screening undertaken in the United Kingdome
Causes of materno-placental problems & their management
Family planing concepts & methodology. Candidates do not
have to have a detailed knowledge of specific drug but
knowledge of drug groups would be beneficial
Management of infertility
Genitourinary problems in women their presentation,
diagnosis, investigation & management
Paediatrics
There is an overlap of this section with other medical
sections ( e.g. coeliac disease )
Common genetic conditions (Down/Edwards etc)
The concept of Mendelian inheritance (AD/AR/XR/XD)
Concept of child abuse (both obvious & occult)
Preventative paediatrics (immunization /health screening
etc)
Psychiatry
The management of psychiatric conditions
When would you refer to a psychiatrist ?
If you managed people in your surgery what cases are best
to managed & when should you refer
What are the facilities available foe community care of
psychiatric patients (there has been a drive to out of the
hospital care of these patients)
Candidates should realize that there is an overlap of
psychiatric patients can have medical conditions)
The diagnosis of psychiatric conditions-the psychiatric history
& mental state examination

Miscellaneous topics
Public health medicine.
Infectious disease control
Immunisation policy & immunisations
Population studies - epidemiology
Statistics (basic concepts only)

Jen & Adil

You might also like